Location via proxy:   [ UP ]  
[Report a bug]   [Manage cookies]                

Document (Recovered)

Download as pdf or txt
Download as pdf or txt
You are on page 1of 141

Esophagus and Stomach

Question 1 A 56-year-old physician attends your office the day after an upper endoscopy performed for
dyspeptic symptoms. He has already read the report, which mentions diffusely thickened gastric folds.
He is very worried and asks about the potential differential diagnosis.

Which of the following is not a differential diagnosis for thickened gastric folds?

A. Zollinger–Ellison syndrome

B. Pancreatic rest

C. Gastric lymphoma

D. Menetrier's disease

E. H. pylori gastritis

Sorry, you have selected the wrong answer.

ANSWER: B. Thickened gastric folds can represent a local malignant process or an inflammatory or
hormonal response. In Zollinger–Ellison syndrome, the hypergastrinemia leads to parietal cell
hyperplasia and prominent folds. Menetrier's disease, a rare protein-losing hypertropic gastropathy is
most commonly seen in middle-aged men, and is a diagnostic possibility in this case. H. pylori gastritis
can significantly alter gastric folds, and can mimic malignancy in severe cases. Pancreatic rest usually
presents in the antrum as a single subm ucosal lump, and does not cause diffusely enlarged gastric folds

Question 2 A 79-year-old man is evaluated for epigastric pain, loss of appetite, and early satiety.
Upper endoscopy reveals a submucosal mass in the gastric antrum. EUS shows a 4 × 6 cm mass in the
mid stomach arising from the second layer (mucosa/lamina propria).

On the basis of the endosonographic appearance, which tumor is most likely?

A. Carcinoid tumor

B. Lipoma

C. Pancreatic rest

D. GIST

E. Duplication cyst

Sorry, you have selected the wrong answer.

ANSWER: A. Carcinoid tumors arise from the second layer (mucosa/lamina propria) and can cause all of
these symptoms. Lipomas are usually asymptomatic, and arise from the third layer (submucosa).
Pancreatic rest usually arises from the third or fourth layer (muscularis propria). Most GISTs arise from
the fourth layer. Duplication cysts are usually asymptomatic and arise from the third layer, or can even
be extrinsic to the GI wall.

Question 3 A 56-year-old man attends for review of his Barrett's esophagus (BE). He was diagnosed 8
years ago with short-segment non dysplastic Barrett's esophagus, (NDBE), which was confirmed 12
months later. He had endoscopies 5 and 2 years ago, which continued to show NDBE.
He asks, of 1000 patients in my scenario, how many develop esophageal adenocarcinoma (EAC) each
year?

A. 1–3

B. 5–10

C. 10–15

Question 3 A 56-year-old man attends for review of his Barrett's esophagus (BE). He was diagnosed 8
years ago with short-segment non dysplastic Barrett's esophagus, (NDBE), which was confirmed 12
months later. He had endoscopies 5 and 2 years ago, which continued to show NDBE.

He asks, of 1000 patients in my scenario, how many develop esophageal adenocarcinoma (EAC) each
year?

A. 1–3

B. 5–10

C. 10–15

D. 20–50

E. Close to 100

Sorry, you have selected the wrong answer.

ANSWER: A. Major studies show that progression rates of NDBE to EAC are low, particularly when NDBE
is confirmed on several biopsies. The best current estimate is that 0.14–0.30% of patients with NDBE
develop EAC per year.
Question 4 A 27-year-old woman with idiopathic gastroparesis diagnosed 6 months ago is hospitalized
with dehydration from persistent nausea and vomiting. She is now unable to keep down solid foods and
has limited tolerance of liquids. She has lost 20 lb in the past month (baseline weight 132 lb, height 5′
6″). Prior endoscopy with gastric biopsy was normal. An upper GI barium study showed delayed gastric
transit with normal duodenal transit and no obstruction. Her recent abdominal X-ray is normal.

What method should be used to provide nutrition support?

A. Parenteral nutrition

B. Gastrostomy tube

C. High-fat diet

D. Jejunostomy tube

E. None – nutrition support is not yet indicated

Sorry, you have selected the wrong answer.

ANSWER: D. The patient has failed oral nutrition and is severely malnourished. Enteral feeding is the
preferred method of nutrition support if the gut is functional. Her prior endoscopy, barium study, and
current abdominal X-ray show no evidence of small bowel or colonic ileus to indicate a diffuse
dysmotility syndrome. A trial of nasojejunal tube feeding should be given if there is doubt as to whether
the small bowel is functional. A high-fat diet delays gastric emptying and would worsen her symptoms.

Question 5 A 60-year-old man who has Prague classification C6M8 Barrett's esophagus and a single
nodule that is removed with endoscopic mucosal resection is found to have high-grade dysplasia. The
remainder of the esophagus is non-dysplastic. RFA of his remaining Barrett's esophagus is
recommended. Following complete ablation, which takes three RFA treatments, he appears to have
complete remission from intestinal metaplasia.

After 5 years, what is the approximate likelihood of recurrent Barrett's esophagus in this patient?

A. 5%

B. 25%

C. 50%

D. 75%

E. 85%

Sorry, you have selected the wrong answer.

ANSWER: B. The 5-year recurrence of Barrett's esophagus is currently estimated to be 15–35%, with
newer studies reporting lower numbers. Most recurrences are non-dysplastic and amenable to further
endoscopic therapy.

Question 6 A 35-year-old man recently underwent laparoscopic myotomy with 180° fundoplication for
achalasia. Postoperatively, his dysphagia has improved, but the patient still has to eat slowly and
continues to regurgitate twice weekly. His weight has remained stable.

After 4 months of persistent symptoms, a barium esophagogram reveals marked improvement in


esophageal emptying compared to preoperatively, but some delay in emptying of liquids. An esophageal
manometry reveals a basal mid-respiratory LES pressure of 11 mmHg.

What is the best treatment option for this patient's persistent symptoms?

A. POEM

B. LES injection with botulinum toxin

C. Pneumatic dilation

D. Through-the-scope balloon dilation to 20 mm

E. Dietary modification

Sorry, you have selected the wrong answer.

ANSWER: D. The manometry is not worrisome for incomplete myotomy at this stage. The first and safest
option would be to proceed with a 20 mm dilation. If ineffective, a pneumatic dilation or POEM could be
considered. Dietary modification is not an appropriate step here as there are interventions which might
lead to symptom improvement. Injection of botulinum toxin is completely inappropriate in this scenario.

Question 7 A 54-year-old Caucasian male with dyspepsia that has persisted despite starting omeprazole
20 mg daily 3 months ago presents for EGD. He takes metoprolol 40 mg daily for hypertension and is
otherwise in good health. A screening colonoscopy 3 years ago was normal, and he has never had a
previous EGD. His family history is negative for any GI illness. He denies smoking or alcohol abuse, and
has never traveled abroad.

During the EGD, multiple subcentimeter (between 5 and 7 mm) polyps are observed in the body and
fundus of the stomach. The rest of the exam is normal.
What is the most appropriate next step?

A. Discontinue the omeprazole, as it is most likely the cause of these polyps

B. Repeat EGD in 1 year, as these polyps are smaller than 1 cm

C. Biopsy the polyps and the surrounding normal mucosa for Helicobacter pylori

D. Perform an EUS as it will assist in diagnosis of these lesions

E. Biopsy the polyps only

Well done, you have selected the right answer.

ANSWER: C. It is essential to exclude gastric adenomas in this case, hence biopsy is essential. Long term
PPI use can lead to fundic gland polyps but in this case the exposure is short. Current guidelines
recommend biopsing gastric polyps, removing polyps that are adenomas (or those with dysplasia) and
eradicating H. pylori if present. Fundic gland polyps and hyperplastic polyps account for 90% of cases
with gastric polyps, and malignant transformation in these cases is rare (unless >1cm and even then the
rate is only about 1%).

Question 8 A 39-year-old male with dyspepsia who is otherwise in good health presents for EGD. He
does not take any over-the-counter or prescription medications, is a social drinker, and denies smoking.
Both of his parents are in their 70s and in good health, and he has one brother, who was diagnosed with
diverticulosis last year at 45 years of age.

During the EGD, multiple polyps are seen distributed in a carpet-like pattern covering the body of the
stomach. They range in size from 5 to 10 mm. Several are resected and the pathology returns as fundic
glands without dysplasia. Helicobacter pylori is negative.

What is the most appropriate next step?


A. No testing is needed as no dysplasia has been determined

B. Repeat EGD in 1 year for surveillance

C. Repeat EGD in 3 years for surveillance

D. Capsule endoscopy, to assess for small-bowel polyps

E. Colonoscopy

Sorry, you have selected the wrong answer.

ANSWER: E. The concern here is FAP (familial adenomatous polyposis). In FAP, the majority (over 50%)
have fundic gland polyps, with dysplasia often being found (40%). This patient is not on a PPI and is
unlikely to have pernicious anemia which may cause multiple benign polyps in the stomach.

Question 9 A 60-year-old woman presents with symptoms of regurgitation and bloating for the past 3
years. She denies heartburn, weight loss, and dysphagia. Her travel history is negative, and no new
medications have recently been introduced. Her past health is significant for depression and type 2
diabetes, with evidence of a peripheral neuropathy. She currently takes citalopram, metformin, and
omeprazole, as well as hydrocodone for daily headaches. Her latest hemoglobin A1c is 10.2%. Physical
examination reveals an elevated BMI of 28 and mildly decreased sensation to vibration in both feet
below the knees. No succussion splash is heard during physical examination. An EGD was normal 12
months ago.

What is the most appropriate next step?

A. Perform ambulatory pH monitoring


B. Perform a 4-hour gastric-emptying study

C. Perform a stool test for Helicobacter pylori antigen

D. Perform esophageal manometry

E. Advise the patient to stop hydrocodone

Sorry, you have selected the wrong answer.

ANSWER: E. This diabetic patient has symptoms possibly consistent with gastroparesis, but opiate
medications can delay gastric emptying and interfere with accurate assessment of gastric emptying. A
gastric-emptying study would be appropriate if the symptoms persist after opiate cessation.

Dyspeptic symptoms can occur in patient with Helicobacter pylori, but the symptom benefit of
eradication is small. Furthermore, PPIs interfere with H. pylori stool antigen testing, and are therefore
not the next logical step. In the absence of dysphagia, an esophageal manometry is unlikely to be
helpful. Ambulatory pH monitoring is likely to show reflux in this patient, but the underlying
pathophysiology of delayed gastric emptying needs to be explored by first stopping opiates and then
assessing gastric emptying.

Question 10 A 60-year-old attorney presents to clinic with heartburn symptoms that have become
refractory to 20 mg over-the-counter omeprazole over the past 4 months, which he has used 1–2 days
per week for the past 15 years. He denies regurgitation, but mentions that bread and meat have
become more difficult to swallow over the past 2 months. His past health is significant for renal calculi,
gout, and hypertension. He stopped all tobacco use on his 50th birthday.

What is the most appropriate next step?

A. Refer for upper endoscopy

B. Perform ambulatory pH monitoring, due to refractory symptoms


C. Perform a gastric-emptying study

D. Perform esophageal manometry

E. Increase the omeprazole to 40 mg twice daily

Well done, you have selected the right answer.

ANSWER: A. This patient should undergo an upper endoscopy, as dysphagia is a red-flag symptom for
esophageal cancer. Furthermore, his age, gender, and prior smoking history put him at higher risk for
upper GI neoplasia. A manometric study or pH impedance study could be considered if the EGD does not
show a structural cause for dysphagia. An increase in PPI medication without an endoscopic evaluation
could potentially lead to diagnostic delay, and clinical trials do not support increased efficacy of
twice-daily PPI over once-daily.

Question 11 A 45-year-old woman with a past medical history of hyperthyroidism and rheumatoid
arthritis presented 6 months ago with melena. An EGD and colonoscopy at that time were negative. She
now presents with another episode of melena and hemoglobin of 7 g/dL. She is hemodynamically
improved after crystalloid resuscitation.

What is the most appropriate next step?

A. Capsule endoscopy

B. Tagged RBC scan

C. Repeat EGD and colonoscopy

D. Device-assisted enteroscopy
E. CT angiography

Sorry, you have selected the wrong answer.

ANSWER: C Due to the high rate of missed lesions at the index endoscopies a repeat EGD and
colonoscopy should be performed. Therefore proceeding directly to capsule endoscopy at this time is
incorrect. CT angiography and tagged RBC scans can be helpful in cases of obscure GI bleed, but would
not achieve as high a yield in this patient at this point in time as the standard EGD and colonoscopy. A
device-assisted enteroscopy should be reserved for patients in whom a potential treatable target has
been seen on a capsule endoscopy or other imaging modality.

Question 12 A 65-year-old man presents with GERD in the setting of obesity. His current BMI is 38
kg/m2. He has T2DM, hypertension, and heart failure. He explains that ever since childhood, he has had
an above-normal appetite.

Which of the following gut peptides does not play a role in appetite suppression?

A. Cholecystokinin

B. GLP-1

C. Ghrelin

D. Peptide tyrosine tyrosine

E. Pancreatic polypeptide

Sorry, you have selected the wrong answer.

ANSWER: C. Cholecystokinin, peptide tyrosine tyrosine, pancreatic polypeptide, and GLP-1 all delay
gastric emptying and thus suppress appetite. Ghrelin, however, increases gastric motility and stimulates
release of growth hormone, thus promoting appetite.
Question 13 A 37-year-old man is referred to the GI clinic for a second opinion on his eosinophilic
esophagitis (EoE). He was diagnosed more than 9 years ago after several presentations to ER with food
bolus obstruction. Endoscopy following a 2-month course of high-dose PPI therapy revealed a
significantly elevated eosinophil count, of >60 eosinophils per HPF in all biopsies from the esophagus,
and normal gastric and small biopsies. He was commenced on oral viscous budesonide and repeat
endoscopy revealed resolution of eosinophilia. He has noted ongoing dysphagia, and needs to wash
food down with plentiful liquid during meals. It takes him a long time to finish a standard meal, causing
embarrassment at restaurants.

What is the most appropriate next step?

A. Call his pharmacy: he is probably not taking his medicines

B. Double his current oral steroid dose

C. Start systemic corticosteroid therapy

D. Recommend a six-food elimination diet

E. Perform an esophagram

Sorry, you have selected the wrong answer.

ANSWER: E. This patient at this stage has documented resolution of his esophageal eosinophila but has
persistent symptoms. This is most likely due to submucosal fibrosis, and an esophagram is more
accurate than endoscopy at providing an accurate estimate of the caliber of his esophagus. His
symptoms are not likely to resolve until he has this addressed with dilation via EGD, and the
esophagram will help determine focal strictures versus a diffusely narrowed (small-caliber) esophagus.
Doubling the dose of steroids or commencing systemic steroids is not going to help, as this patient
already has developed control of the eosinophilia. Likewise, a six-food elimination diet is an acceptable
therapy for EoE, but the cause of dysphagia in this cause is not lack of response to therapy. Medication
non-compliance can be a cause of treatment failure, but in this case an esophagram is the best next
step.
Question 14 A six-food elimination diet is an acceptable therapy for eosinophilic esophagitis (EoE).
Which of the following foods does not form part of the standard six-food elimination?

A. Eggs

B. Fish and shellfish

C. Soy and milk

D. Nuts and wheat

E. Meat

Sorry, you have selected the wrong answer.

ANSWER: E. A standard six-food elimination involves systematic elimination of eggs, fish, and shellfish
soy, nuts, wheat, and milk, with gradual introduction followed by repeated endoscopy at each step to
assess response. Meats are not part of the standard elimination diet

Question 15 A 30-year-old man describes daily postprandial heartburn and regurgitation for 6 months. It
is particularly bothersome on weekends, and has persisted despite stopping coffee and alcohol. He
denies dysphagia or weight loss. His only history is asthma. Physical examination reveals a
healthy-appearing young man who is not overweight.

What is the most appropriate next step?

A. Initiate omeprazole 40 mg twice daily

B. Perform a 48-hour wireless pH test


C. Initiate omeprazole 20 mg daily

D. Initiate ranitidine 150 mg before bedtime

E. Refer for laparoscopic fundoplication

Sorry, you have selected the wrong answer.

ANSWER: C. A therapeutic trial of once-daily standard-dose PPI would be appropriate, with symptom
improvement considered diagnostic of GERD, and no further evaluation would be indicated. pH testing
might be useful if no PPI response is seen. H2 receptor antagonists are not as effective as PPIs in
controlling symptoms of GERD long-term, but may be a reasonable alternative in select cases. Referral
for laparoscopic fundoplication is not appropriate without formal evaluation of the severity of GERD and
esophageal manometry.

Question 16 A 30-year-old man presents with abdominal pain, nausea, and intermittently irritated and
red eyes. The symptoms have been present over the past 12 months, but have increased in intensity. His
past health is significant only for cigarette smoking and a traumatic ankle fracture. He reports no
overseas travel and does not take any regular prescribed or over-the-counter medicines. Physical
examination reveals mild, generalized abdominal tenderness. His primary physician referred him for an
EGD, which looked macroscopically normal except for some gastric erosions, which were biopsied.

In this clinical scenario, the presence of only increased intraepithelial lymphocytes in biopsies from
gastric epithelium should make one consider which of the following?

A. Helicobacter pylori

B. Sarcoidosis

C. Crohn's disease

D. Ulcerative colitis
E. Gastric cancer

Sorry, you have selected the wrong answer.

ANSWER: C. Inflammatory bowel disease should be considered in a young man with abdominal pain, and
Crohn's disease can present with lymphocytic gastritis and iritis. Excess gastric lymphocytosis is a marker
for celiac disease (or drugs) in the stomach, and celiac disease should be excluded. Lymphocytic gastritis
presents with nodules and erosions. Celiac disease also causes intraepithelial lymphocytosis in the small
intestine. Gastric cancer is rare in young men, and will usually be seen macroscopically during an EGD at
the time of symptomatic presentation. Sarcoid can cause granulomatous gastritis. H. pylori is unlikely to
explain intraepithelial lymphocytes and would not explain his symptoms.

Question 17 A 36-year-old teacher presents with regurgitation after meals. She describes belching up
food within 10–20 minutes of most meals, and will often spit the regurgitated material out. She is
gaining weight. A prior evaluation with a 48-hour pH test showed marked esophageal acid exposure in
the upright position, but normal acid parameters during sleep. Her past health is significant for Graves'
disease and a pulmonary embolus after delivery of her second child.

What is the most appropriate next step?

A. Commence high-dose PPI therapy

B. Refer for laparoscopic fundoplication

C. Commence metoclopramide prior to meals

D. Refer for behavioral management

E. Perform a gastroduodenal manometry


Sorry, you have selected the wrong answer.

ANSWER: D. This patient most likely has rumination syndrome. The mainstay of therapy is behavioral
management with diaphragmatic breathing exercises. High-dose PPI therapy is unlikely to help and may
make symptoms worse. Antireflux surgery is not recommended and and metoclopramide is unlikely to
help as this patient has no symptoms of gastroparesis. Rumination is a clinical diagnosis and
postprandial high resolutionesophageal manometry or gastroduodenal manometry can be confirmatory,
but these tests are somewhat invasive and not widely available.

Question 18 A 35-year-old woman presents with symptoms of postprandial regurgitation and bloating
for 12 months. She denies heartburn, weight loss, and dysphagia. She does not report significant
vomiting. Her travel history is negative, and no new medications have recently been introduced. Her
past health is significant for depression and Graves' disease. She currently takes escitalopram. Physical
examination reveals no acute findings. A 90-minute gastric-emptying test shows delayed emptying, and
the patient is told she has gastroparesis. Metoclopramide 10 mg tab t.i.d. has not helped her symptoms.

What is the most appropriate next step?

A. Change metoclopramide to liquid form to improve absorption

B. Perform a 4-hour gastric-emptying study

C. Change to liquid erythromycin

D. Refer the patient for placement of a jejunostomy tube for enteral feeding

E. Institute a liquefied diet

Sorry, you have selected the wrong answer.

ANSWER: B. This is a common clinical scenario, where a diagnosis of gastroparesis has been made on a
short-duration gastric-emptying study, which is frequently unreliable. A four hour study is needed for
confirmation. The lack of vomiting and weight loss makes severe gastroparesis less likely, and thus
placement of a feeding tube is inappropriate. Any change in medication is not indicated until the
diagnosis has been confirmed. Dietary management is the mainstay of therapy for gastroparesis, and
many patients with delayed gastric emptying can maintain adequate oral intake with a liquefied diet.

Question 19 A 23-year-old man presents with increasing dysphagia to solids and liquids. He has choking
attacks occurring at night when supine in bed. His symptoms have been present for 8 months, and he is
now losing weight. He is avoiding most solids and relies on a liquid diet. He feels he can't belch after a
meal. His past health is significant for a knee reconstruction and asthma. An EGD performed 4 months
ago appeared normal, though some food was seen in the stomach. A CT scan of the chest and abdomen
ordered by his primary physician was normal. A gastric-emptying test showed delayed emptying at 90
minutes, but prokinetic therapy has not helped his symptoms.

What is the most appropriate next step?

A. Perform an esophagram and esophageal manometry

B. Perform a 4-hour gastric-empting study

C. Place a feeding tube for enteral feeding

D. Perform a pH impedance study

E. Inject Botox into the gastroesophageal and prepyloric region

Sorry, you have selected the wrong answer.

ANSWER: A. Dysphagia to solids and liquids with associated weight loss is highly suggestive of achalasia.
A diagnosis of gastroparesis is often falsely made, as the holdup of food in the esophagus falsely affects
the study, hence option B is incorrect. The best way forward is to perform an esophagram and
manometry. Endoscopic findings in achalasia are often subtle and are easily missed. Placing a feeding
tube is inappropriate. Empirically injecting Botox to treat possible achalasia and/or gastroparesis
without a diagnosis is discouraged.

Question 20 A 68-year-old woman presents with weight loss and increasing dysphagia to solids and
liquids 18 years after a Heller myotomy with anterior partial fundoplication for achalasia. She did well
after the initial surgery and gained significant weight over the next 12 months. She lives in a remote area
and has not sought attention for her increasing symptoms, which have come on very gradually over the
past 7 years. She is otherwise in good health, though her BMI is now 19. An esophagram shows a dilated
esophagus (9 cm) with a sigmoid configuration. The lowest portion of the esophagus lies below the GEJ,
where a typical bird-beak configuration is seen.

What is the most appropriate next step?

A. Perform an esophagectomy

B. Repeat Heller myotomy

C. Perform pneumatic dilatation with a 30 mm balloon

D. Inject Botox into the GEJ

E. Perform POEM

Sorry, you have selected the wrong answer.

ANSWER: A. This patient has end-stage achalasia. The sigmoid deformity with a dependent portion
makes the likelihood of any GEJ-based therapy (options B–E) very unlikely to work. The patient should
be referred to a surgeon for esophagectomy at a center with high-volume experience.

Question 21 A 48-year-old woman attends asking about the risk factors for Barrett's esophagus, with a
view to potentially modifying them. Her father has just been found to have a 2 cm segment of Barrett's
esophagus, with repeat biopsies confirming the presence of goblet cells and no dysplasia, and she wants
to avoid this in herself.

Which of the following is not considered a risk factor for development of Barrett's esophagus?
A. Alcohol use

B. Central obesity

C. Cigarette smoking

D. Metabolic syndrome

E. Absence of H. pylori infection

Sorry, you have selected the wrong answer.

ANSWER: A. Alcohol has not been found to be an independent risk factor for Barrett's esophagus. All the
others increase the risk.

An inverse relationship is observed between H. pylori infection and Barrett's esophagus. However, the
“protection” against GERD provided by H. pylori-induced atrophic gastritis is not outweighed by the
gastric carcinogenic potential, and thus H. pylori eradication is still recommended in patients at risk of
Barrett's esophagus, if otherwise indicated.

Question 22 A 17-year-old female presents to the emergency room with sudden-onset odynophagia and
dysphagia. She noticed similar symptoms 2 months ago, but they are now much worse, and she can
barely swallow anything. Past medical history is significant for acne vulgaris and iron-deficiency anemia
due to menorrhagia, for which she has received doxycycline, oral iron supplementation, and vitamin C
tablets. She also takes occasional aspirin for headaches Physical examination reveals an anxious young
lady with normal vital signs. There is no crepitus in the neck and no abdominal findings, and the oral
cavity appears normal. A chest X-ray is within normal limits. Pill esophagitis is suspected.

Which of the following medications is least likely to cause pill esophagitis?

A. Doxycycline
B. Enteric coated aspirin

C. Oral iron tablet

D. Vitamin C tablet

E. Oral magnesium tablet

Sorry, you have selected the wrong answer.

ANSWER: E. Over half of all cases of pill esophagitis are caused by antibiotics, and doxycycline is among
the most common, perhaps in part due to the tablet size. NSAIDs, including aspirin, oral iron tablets, and
vitamin C tablets, have been reported to cause pill esophagitis, but no cases have ever been reported
with magnesium tablets. Though not relevant in this patient, another important medication class to
consider is the oral bisphosphonates.

Question 23 A 19-year-old college student presents with gastroesophageal reflux symptoms and
constant belching. These symptoms have been presents for several months and are interfering with his
quality of life. An upper endoscopy and barium esophagogram do not show any visible abnormality. A CT
scan of the abdomen is benign. In the office, the patient belches every 2–3 minutes during the
interview, while in his own home the frequency is approximately every 5–10 minutes.

Which strategy is most likely to be of diagnostic value?

A. Advise the patient against drinking carbonated beverages

B. Advise the patient not to use chewing gum

C. Advise the patient to only drink using a straw


D. Advise the patient to try an enzymatic mouthwash

E. Advise mouth opening for several minutes during the interview

Sorry, you have selected the wrong answer.

ANSWER: E. This patient presents with classic aerophagia and may have supragastric belching. The most
important aim of the patient–physician interaction is to establish the diagnosis. Prolonged mouth
opening in the office usually arrests the belching behavior virtually immediately due to prevention of
swallowing. Avoiding carbonated beverages is usually beneficial in patients with excessive belching.
Patients with excessive belching should not be allowed to drink with a straw or chew gum as this
promotes aerophagia. An enzymatic mouthwash may help if the excessive swallowing is caused by dry
mouth.

Question 24 Two antral and one corpus biopsy of the gastric mucosa are taken during endoscopy of a
55-year-old man with occasional dyspepsia. A lesser curve gastric ulcer is found. The pathology report
shows chronic active gastritis, with one specimen showing intestinal metaplasia. Endoscopy is scheduled
8 weeks after Helicobacter pylori eradication to confirm healing of the gastric ulcer, assess the gastric
cancer risk, and decide whether surveillance is necessary.

Which method should be used to stage for cancer risk?

A. OLGA or OLGIM system

B. Sydney system

C. Assess acute and chronic gastritis activity.

D. Subtyping the type of intestinal metaplasia

E. Degree of carditis.
Well done, you have selected the right answer.

CORREECT A

Question 25 A 25-year-old woman presents with near-constant heartburn, which has not responded to
PPI therapy. She is very anxious about the results of a prior endoscopy, which showed an irregular Z-line.
No esophagitis was noted, but biopsies demonstrated intestinal metaplasia without goblet cells. No
dysplasia was noted.

What is the most appropriate next step?

A. Offer surveillance for Barrett's esophagus

B. Offer ablative therapy for Barrett's esophagus

C. Reassure the patient that she does not have Barrett's esophagus

D. Discontinue surveillance, if a repeat endoscopy confirms absence of goblet cells

E. Discuss the role of prophylactic esophagectomy to prevent cancer development

Sorry, you have selected the wrong answer.

ANSWER: D. The presence of goblet cells is required to make a diagnosis of Barrett's esophagus,
according to US guidelines. If two consecutive biopsies confirm the absence of goblet cells then further
surveillance is not indicated. Thus, option A is only partially correct, as the patient does not fulfill current
diagnostic criteria for Barrett's esophagus. Option C is correct if a second biopsy confirms the absence of
goblet cells. Options B and E are completely inappropriate in this patient.

Question 26 A 65-year-old Caucasian man is diagnosed with long-segment Barrett's esophagus, which is
circumferential in the lower 4 cm of his esophagus and has a maximal extent of 7 cm. No raised lesions
are seen. Several of the biopsies demonstrate low-grade dysplasia, and in the lower 2 cm a single focus
of high-grade dysplasia is noted. After discussing the options for therapy at this stage, the patient elects
to go ahead with RFA of his entire segment of Barrett's esophagus. After three sessions of therapy, no
residual evidence of Barrett's esophagus is noted.

Which of the following statements is correct regarding the need for ongoing surveillance?

A. This patient has complete remission of his Barrett's esophagus and should only have surveillance if
new symptoms develop at annual review

B. Endoscopic surveillance should be performed at 2-yearly intervals indefinitely

C. Endoscopic visualization is required, but biopsies should only be taken if Barrett's esophagus recurs

D. Endoscopic surveillance is indicated, and the prior extent of Barrett's esophagus should be biopsied

E. The patient should be reassured and no follow-up is required

Sorry, you have selected the wrong answer.

ANSWER: D. Despite complete remission, further surveillance is recommended, as the 3–5-year


recurrence of Barrett's is estimated to be 20–40%. Many clinicians initially use a 3-month interval for
surveillance, but this very intensive regimen is not continued indefinitely. Biopsies of the entire prior
length of Barrett's esophagus should be taken during surveillance.

Question 27 A 58-year-old female presents for EGD and EUS for a nodule that was seen by another
gastroenterologist last week. The original EGD was performed for heartburn, and biopsies of the nodule
are not available. The patient was recently diagnosed with type 2 diabetes, was started on enalapril,
simvastatin, and metformin 3 weeks ago by her primary care physician, and has had watery diarrhea.
Her history is otherwise significant for an iron-deficiency anemia.

During the procedure, a 2 cm smooth submucosal nodule is seen in the antrum. As biopsies are taken
with a cold forceps, the nodule feels firm. Under EUS, the lesion appears hypoechoic, with well-defined
borders arising from the muscularis propria.

What is the most likely diagnosis?

A. Lipoma

B. GIST

C. Gastric carcinoid

D. Pancreatic rest

E. Granular cell tumor

Well done, you have selected the right answer.

ANSWER: B. Gastrointestinal stromal tumours (GISTS) have a characteristic EUS pattern – hypoechoic
mass arising from the muscularis propria (4th layer the majority) or the muscularis mucosae (2nd layer –
some small lesions). Predictors of a benign GIST include smaller size (<3cm) and a regular margin as in
this case. Lipomas are hyperechoic, arise from the submucosa and display the “pillow or cushion sign”
(indents when probed). A carcinoid may be hypoechoic but is rare and arises from the mucsoa or lamina
propria. Granular cell tumours are rare, hypoechoic and arise from the mucosa or submucosa. A
pancreatic rest is hypoechoic – the nodule is submucosal with a central umblication.

Question 28 Which is the most important step in gastric acid secretion?

A. Parietal cell stimulation by gastrin

B. Enterochromaffin cell stimulation by gastrin


C. Parietal cell stimulation by acetylcholine

D. Inhibition of somatostatin

Sorry, you have selected the wrong answer.

ANSWER: B. The parietal cells possess receptors for histamine, acetylcholine, and gastrin, and activation
of all three receptors can stimulate acid secretion. However, the most important step for acid release is
the stimulation of the ECL cell to secrete histamine.

Question 29 A 52-year-old man presents with retrosternal chest pain, shortness of breath, and
dysphagia of 24 hours' duration. He has a background of intermittent dysphagia for solids. Prior EGD 2
years ago showed a benign lower esophageal stricture that was dilated. On arrival at the emergency
department, examination revealed left lung base crepitation but was otherwise unremarkable. He has
been started on IV fluids and admitted.

What is the most appropriate next step?

A. Urgent EGD for removal of food bolus

B. Monitoring for further resolution of symptoms

C. Urgent CT scan and a surgical review

D. Broad-spectrum antibiotics

E. Glucagon

Sorry, you have selected the wrong answer.


ANSWER: C. The clinical scenario is suggestive of complete food bolus obstruction. This is probably
complicated by an esophageal perforation, as evident from shortness of breath and left-sided
crepitation resulting from pleural contamination. Doing an urgent gastroscopy and placing endoclips or
luminal stents is an option. However, by itself this is unlikely to help if there is a significant
contamination of the pleural cavity, as in this patient.

Monitoring and non-operative intervention is only acceptable in highly select patients with contained
perforations who are clinically stable and otherwise asymptomatic. Administration of glucagon (1 mg IV)
can be attempted to relax the esophagus; this may promote passage of the food bolus. Studies of
glucagon in this setting have had mixed results. It can be administered in uncomplicated food bolus
obstruction, but this should not delay definitive endoscopic therapy. While the patient will need
broad-spectrum antibiotics to treat the infection resulting from plural cavity contamination, this is
unlikely to be of benefit on its own.

Further imaging with CT scan or X-ray may reveal a radio-opaque foreign body or signs of esophageal
perforation, such as mediastinal, subdiaphragmatic, or subcutaneous air. An operative intervention
following this is the safest approach and is generally considered the treatment of choice, particularly for
those seen more than 24 hours after the onset of symptoms.

Question 30 A 24-year-old male presents with acute dysphagia, restrosternal chest pain, and fever. He
was earlier diagnosed with eosinophillic esophagitis and was treated with swallowed fluticasone. On
presentation, WBCs are 11.2 cells/mm3, with raised CRP. Gastroscopy reveals severe esophagitis with
well-circumscribed ulceration with a central crater. Histology shows multinucleated giant cells, with
ground-glass nuclei and eosinophilic inclusions.

What is the most appropriate next step?

A. Acyclovir

B. Fluconazole

C. Pantoprazole
D. Stop fluticasone

E. Gancyclovir

Sorry, you have selected the wrong answer.

ANSWER: A. The gastroscopy and the histology findings confirm the diagnosis of HSV esophagitis. The
patient has been on long-term swallowed fluticasone, which is a risk factor in the absence of any other
immunocompromised state. Stopping fluticasone is recommended, but by itself is not going to alter the
clinical course. Acyclovir should be promptly administered.

CMV infection is another important differential in this patient. The ulcers related to CMV esophagitis are
linear or longitudinal and are deep. This histology shows vascular endothelial cell damage with the
presence of intranuclear and intracyctoplasmic inclusions and characteristic cytomegalic cells.
Gancyclovir is used for treatment.

Fluconazole is effective for candida esophagitis and pantoprazole for reflux esophagitis. Both are
unlikely to help in this case.

Question 31 A 19-year-old obese man is referred to the GI clinic after he presented to the emergency
room complaining of a piece of chicken was stuck in his esophagus after dinner. His symptoms resolved
while in the waiting room and he tolerated oral liquids, so he was discharged without further
investigations. He admits to similar, albeit less dramatic symptoms over the past few years. He denies
ever having dysphagia to liquids. His medical background is only significant for exercise-induced asthma.
An EGD appears macroscopically fairly unremarkable, but histopathology confirms the presence of
eosinophilic infiltration, with a count of 25 per HPF in his distal esophagus and 8 per HPF in the mid
esophagus.

What is the most appropriate next step?

A. Commence swallowed corticosteroid therapy

B. Commence high-dose PPI therapy


C. Refer for skin testing to identity food allergens

D. Recommend a six-food elimination diet

E. Defer further management until after an esophageal manometry

Sorry, you have selected the wrong answer.

ANSWER: B. This patient at this stage has symptomatic esophageal eosinophilia (EE). A diagnosis of
eosinophilic esophagitis (EoE) cannot be made until a trial of high-dose PPIs for 8 weeks has been
undertaken (if he responds, he will be labeled PPI-responsive esophageal eosinophilia). Option A and D
are appropriate for patients with EoE not responding to PPI. Skin allergy testing is currently of limited
clinical usefulness. An esophageal manometry is not required in a young healthy man with esophageal
eosinophilia.

Question 32 A 58-year-old woman presents with progressive dysphagia and weight loss. Multiple prior
endoscopies have noted friable esophageal mucosa with a proximal esophageal stricture. No malignant
cells have been noted on biopsies of the esophagus, and a CT scan of the chest and abdomen is
negative. The patient also complains of dyspareunia and mouth ulcers.

Which of the following findings on histology would be most suggestive of a unifying diagnosis?

A. Civatte bodies

B. Esophageal eosinophilia

C. Intestinal metaplasia with goblet cells

D. Squamous parakeratosis
E. Squamous cells with CMV inclusions

Well done, you have selected the right answer.

ANSWER: A. This woman most likely has lichen planus with esophageal, oral, and genital involvement.
Lichen planus is a rare but likely underrecognized disease. The finding of Civatte bodies – necrotic
keratinocytes – is diagnostic, but is rare on esophageal biopsies, and thus in most cases a clinical
diagnosis is made. Option B can cause dysphagia and strictures, but EoE would not explain the oral and
genital symptoms. Intestinal metaplasia with goblet cells (Barrett's esophagus) is usually an
asymptomatic condition. CMV esophagitis can cause a friable-appearing esophagus, but does not usually
involve the oral and genital track. Squamous parakeratosis is usually a benign and asymptomatic finding.

Question 33 A 24-year-old woman with an unremarkable medical background is referred for evaluation
of diarrhea and abdominal pain of several months' duration. She has stopped drinking alcohol as this
worsens her symptoms and gives her flushing. Physical examination reveals a mildly enlarged liver and
spleen. Upon scratching her skin, she has a positive Darier's sign. An upper endoscopy shows a small
peptic ulcer.

Which test is most likely to reveal her underlying disorder?

A. Serum gastrin level

B. Urinary salicylate assay

C. Celiac gene susceptibility testing

D. Serum tryptase level

E. Urine and stool porphyrins

Sorry, you have selected the wrong answer.


ANSWER: D. Diarrhea, flushing after drinking alcohol, and a positive Darier's sign (urticaria upon
scratching the skin) are suggestive of mastocytosis, and a serum tryptase is typically elevated. While
some clinical features are shared with Zollinger–Ellison syndrome, a serum gastrin level is not the best
initial test in this patient. The presentation itself would be unusual for celiac disease, and serum testing
is preferred prior to genetic testing. Patients with acute intermittent porphyria present with episodic
abdominal pain and constipation.

Question 34 A 29-year-old musician present with chronic nausea with frequent vomiting, which has
gradually worsened over the last 18 months. While touring, he has frequently presented to emergency
rooms for management of refractory vomiting and nausea. He has lost 2 kg. His past medical history is
significant for IgA nephropathy in childhood. Physical examination, including assessment of the cranial
nerves, is within normal limits. In addition to regular promethazine, he uses marijuana to promote his
appetite. Upper endoscopy is unremarkable. CT scan of the abdomen shows an accessory spleen and no
other findings. Ultrasound of the liver and gallbladder is unremarkable.

What is the most appropriate next step?

A. Proceed with a gastric-emptying study to evaluate for gastroparesis

B. Proceed with an MRI of the brain to assess for a central cause of the vomiting

C. Prescribe mirtazapine

D. Advise marijuana cessation and reassess clinically in 3–6 months

E. Proceed with a HIDA scan

Sorry, you have selected the wrong answer.

ANSWER: D. This patient likely has cannabinoid hyperemesis. Supportive clinical features would include
relief of symptoms with hot baths and showers. Therefore, the initial strategy should focus on marijuana
cessation and clinical review. Proceeding with a gastric-emptying study would not be unreasonable if
this patient did not use THC. A central cause of vomiting is unlikely in this patient given the duration of
symptoms and the lack of neurological signs, so an MRI at this stage is not recommended. A HIDA scan,
whether positive or negative, is unlikely to be helpful as the clinical presentation is not typical for biliary
disease. Question 35 Which of the following differentiates hyperemesis gravidrum from nausea and
vomiting of pregnancy?

A. Vomiting that persists throughout the entire day

B. Vomiting upon arising that occurs more than 5 days per week

C. Vomiting that begins after the 20th week of pregnancy

D. Vomiting with weight loss >5% of pre-pregnancy body weight

E. Vomiting associated with favorable newborn outcomes

Sorry, you have selected the wrong answer.

ANSWER: D. Nausea and vomiting of pregnancy is common but does not involve systemic changes to the
body. Weight loss >5% suggests the inability to keep down any nutritional calories and is consistent with
hyperemesis gravidarum, which needs to be treated more aggressively.

Question 36 A 50-year-old woman is in your office for vague abdominal pains. She has had these for
several years. She has had no weight loss. The patient takes an aspirin intermittently.

The local physician performed some basic tests and measured a gastrin which was 1800 u/L (normal
<150). She is not on PPI's. EGD demonstrates a few erosions in the antrum. Biopsies demonstrate no H
pylori.

What is the most appropriate next step?


A. Perform an EUS to look for a gastrinoma

B. Perform an Octreoscan

C. Perform a secretin stimulation test

D. Perform gastric acid analysis

E. Perform H. pylori serology

Sorry, you have selected the wrong answer.

ANSWER: D. This is unlikely to be a gastrinoma in the absence of significant peptic ulceration; just
erosions would be most unusual in gastrinoma and are probably explained by the aspirin. Hence, a
workup for gastrinoma would be premature so neither A, B or C are the most appropriate next step.
Pernicious anaemia needs to be ruled out and is a much more common explanation of a raised gastrin;
achlorhydria would then be expected. As gastric biopsies were negative for H. pylori, serology is unlikely
to be helpful and if positive would suggest past infection. Further, H. pylori would not explain such a
high gastrin. The vague abdominal pain is more likely to be functional bowel disease in this setting.

Question 37 A 23-year-old man presents with heartburn, intermittent solid food dysphagia, and
abdominal discomfort. His symptoms have gradually worsened over several months, but he has not lost
weight. An EGD is performed and shows some linear furrows in the esophagus and mild antral
erythema. Biopsies from the proximal and distal esophagus demonstrate >60 eosinophils/HPF, and
gastric biopsies show mild reactive gastritis. A peripheral eosinophil count is elevated.

Which of the following statements describes the most likely diagnosis and the most appropriate next
step?

A. This patient has EoE and topical steroid therapy should be commenced

B. This patient has GERD and a pH impedance study should be performed


C. This patient has eosinophilic gastroenteritis and should receive systemic corticosteroid therapy

D. This patient should have further testing for other causes of peripheral eosinophilia

E. This patient should undergo a six-food elimination diet

Sorry, you have selected the wrong answer.

ANSWER: D. The patient scenario is clinically most consistent with eosinophilic gastroenteritis, as
abdominal pain is usually not a presenting feature in adults (though it can be in children). The next
logical step here is to exclude other causes of eosinophilia, such as drugs and parasitic infection. Option
A is thus less likely, and no therapy should be offered until further testing has occurred. Most patients
with eosinophilic gastroenteritis have at least two segments of affected intestine, but option B is
incorrect as our patient has not had small-bowel and colonic biopsies taken. Option C is incorrect as
another differential diagnosis exists and therapy should await further testing. Option E is incorrect as a
six-food elimination diet only has proven efficacy in EoE, but can be trialed in cases of eosinophilic
gastroenteritis only after the diagnosis has been confirmed.
Intestine and Pancreas

Question 1 A 42-year-old African American male presents to the emergency department with acute
severe epigastric abdominal pain. The pain is 10 out of 10 in intensity, worsens with eating, is associated
with nausea/vomiting, and is non-radiating. The patient has a history of HIV diagnosed 4 years ago and is
currently not on treatment. His most recent CD4 count was 75. He was recently evaluated in the
outpatient setting and was started on fluconazole for oral thrush and on pentamidine for PCP
prophylaxis, as he has sulfa allergy. He has a history of alcohol use but has not had a drink for more than
4 weeks. His labs on arrival show hemoglobin 10.3 g/dL, WBC 2.2 cells/mm3, platelet 145,000, sodium
132 mg/dL, creatinine 0.6 mg/dL, total bilirubin 1.2 mg/dL, AST 40 IU/L, ALT 28 IU/L, alkaline
phosphatase 128 IU/L, lipase 4000 IU/mL, lactate 4.6, and normal urine analysis. CT abdomen only
shows evidence of acute interstitial pancreatitis.

What is the likely diagnosis?

A. Alcoholic pancreatitis

B. Drug-induced pancreatitis related to pentamidine

C. Gallstone pancreatitis

D. Perforated peptic ulcer

E. Pancreatic cancer
Sorry, you have selected the wrong answer.

ANSWER: B. The patient has acute severe epigastric pain and elevated lipase, thus meeting criteria for a
diagnosis of acute pancreatitis. He has not been drinking recently, so alcoholic pancreatitis is unlikely,
and given his normal liver tests are normal gallstone pancreatitis is also less likely. Drug-induced
pancreatitis should be considered when evaluating for etiologies, and pentamidine is a known cause of
acute pancreatitis. CT scan does not show any evidence of perforated ulcer or pancreatic cancer.

Question 2 A 72-year-old male has been experiencing epigastric abdominal discomfort that worsens
after eating, progressive weight loss, and greasy stools over the past 3 months. Physical exam is
unremarkable. Labs reveal normocytic anemia with hemoglobin of 10.5 g/dL, normal kidney function,
and normal liver tests. Amylase and lipase are normal. CT scan of the abdomen reveals mass-like lesions
in the head of the pancreas with indistinct boundaries and no vascular invasion, a diffusely abnormal
(“sausage-shaped”) pancreas with delayed enhancement, and a beaded irregular pancreatic duct. The
common bile duct looks normal. No hepatic masses are noted. Serum IgG4 and serum CA19-9 levels are
normal. EUS of the pancreatic mass is negative for malignancy.

What is the most appropriate next step?

A. Surgical consultation for consideration of Whipple's surgery

B. Reimaging in 6 months to assess progression

C. Steroid therapy

D. Pancreatic enzyme replacement therapy for malnutrition

E. Referral to a medical oncologist for initiation of chemotherapy

Well done, you have selected the right answer.


ANSWER: C. The imaging features described in this question are classic for autoimmune pancreatitis
(AIP). An elevated serum IgG4 can serve as a useful test for diagnosis of AIP, but a negative test does not
rule the disease out given the sensitivity of around 75%. Absence of vascular invasion on CT, normal FNA
of the mass, and a normal CA19-9 make a diagnosis of pancreatic adenocarcinoma less likely. Pancreatic
biopsy is the gold standard for diagnosis of AIP, but the pancreas is usually not amenable to core biopsy
and FNA is not helpful in confirming diagnosis. AIP is exquisitely sensitive to steroid therapy, so a
therapeutic trial with prednisone can be initiated in patients with strong suspicion for AIP and negative
workup for pancreatic cancer.

Question 3 A 45-year-old woman diagnosed with chronic idiopathic constipation fails to respond to
increased dietary fiber and prune juice. It is decided to add another agent.

Which of the following acts on the intestinal chloride channels to relieve constipation?

A. Prucalopride

B. Polyethylene glycol

C. Lubiprostone

D. Linaclotide

E. Tegaserod

Sorry, you have selected the wrong answer.

ANSWER: C. Lubiprostone acts on the type 2 chloride channels (CIC-2). Linaclotide is a guanylate
cyclase-C (GC-C) agonist. Prucalopride and tegaserod act on the 5HT4 receptor. Polyethylene glycol is an
osmotic laxative.

Question 4 Which of the following statements is correct regarding the role of the GI tract in weight
regulation?
A. Gastric distension plays a role in meal initiation

B. Ghrelin is produced in response to the presence of nutrients in the stomach and acts as an appetite
suppressant

C. The gut microbiome may play a role in the development of obesity and insulin resistance

D. The gut–brain interaction leading to food intake regulation involves signals carried exclusively by the
vagus nerve

Sorry, you have selected the wrong answer.

ANSWER: C. Gastric distension does not play a major role in meal initiation, but larger fasting gastric
volumes are associated with higher caloric intake during a meal. Ghrelin is not an appetite suppressant.
The gut–brain interaction is mediated by both vagal and sympathetic (splanchnic) pathways. The gut
microbiome is becoming an intense focus of research into the development of obesity. Several animal
models now suggest that the obese phenotype can be “transplanted” using microbiome.

Question 5 A 45-year-old woman presents with concerns about her personal risk of colorectal cancer.
She wants to be advised whether she is at increased risk and what steps she should take to prevent its
development.

Which of the following aspects of her medical history has the least impact on her future risk of
colorectal cancer?

A. Personal history of pancolonic ulcerative colitis in complete remission on 5-ASA therapy

B. Stage IV sigmoid adenocarcinoma diagnosed in her father at age 82

C. BMI of 32.5
D. Personal history of acromegaly

E. Personal history of cholecystectomy

Sorry, you have selected the wrong answer.

ANSWER: B. A personal history of a colorectal cancer in a first-degree relative at age 82 does not
significantly increase an individual's risk of colon cancer. Note that this is not the case if the affected
relative is diagnosed while younger than 60. For such individuals, earlier colorectal cancer screening is
recommended. Obesity, personal history of ulcerative colitis, acromegaly, and prior cholecystectomy are
all recognized risk factors for colorectal cancer.

Question 6 A 23-year-old female presents for evaluation with abdominal pain, diarrhea, and bloating,
which started gradually 3 months ago. In the last month, painful joints (knees, hips) have developed. She
has taken ibuprofen 400 mg twice daily for the last week. She has not travelled outside of the United
States. Her past medical history is significant for bronchial asthma and exercise-induced anaphylaxis. On
physical examination, she has diffuse mild palpation tenderness, is unequivocally Carnett's
sign–negative, and has a normal rectal exam. An upper endoscopy is normal, but on colonoscopy the
distal 7 cm of her ileum is ulcerated and inflamed. The colon appears normal.

Which of the following statements most accurately describes her clinical scenario?

A. She most likely has ileitis due to backwash from ulcerative colitis

B. The clinical scenario is highly suggestive of Crohn's disease, and empiric therapy can be started

C. Tuberculosis must be excluded

D. While Crohn's disease is likely, this clinical presentation is also consistent with Yersinia infection

E. The presentation is most consistent with NSAID-induced enteropathy


Sorry, you have selected the wrong answer.

ANSWER: D. The clinical presentation is consistent with Crohn's disease, but infection from within the
Yersinia genus can cause a similar clinical picture. Unlike with other diarrheal pathogens, the acute
diarrhea can last several weeks, and many patients have ongoing diarrhea for months. Furthermore,
patients with HLA-B27 tissue type are prone to a postinfectious syndrome consisting of reactive
arthropathy mainly affecting the large weight-bearing joints. Ruling this out before starting
Crohn's-directed therapy is therefore important. While NSAIDs can cause enteropathy and changes in
the terminal ileum, the dose and duration in this case make this less likely. Tuberculosis can mimic
Crohn's disease in highly endemic countries. Finally, while backwash ileitis from ulcerative colitis can
cause terminal ileitis, the findings on colonoscopy do not support this in this patient.

Question 7 What is the best approach when treating chronic diarrhea with opiate antidiarrheal drugs?

A. Repeat doses of drug after each bowel movement until diarrhea is controlled

B. Use a large dose of potent opiates (e.g., codeine, morphine, opium) first, then titrate down

C. Use a small dose of potent opiates (e.g., codeine, morphine, opium) first, then titrate up

D. Use low-potency opiates (e.g., loperamide, diphenoxylate) first, on an as-needed basis

E. Use low-potency opiates (e.g., loperamide, diphenoxylate) first, on a scheduled basis

Sorry, you have selected the wrong answer.

ANSWER: E. Symptom-based therapy of chronic diarrhea is often required in the absence of disease
specific options. Symptom-based therapy should aim to prevent diarrhea, not treat after diarrhea breaks
through (so A and D are incorrect). Codeine, morphine or opium are not optimal because of the risks of
dependency and abuse, albeit uncommon. Low-potency opiates tht do not risk abuse or dependency,
and can be given regularly to prevent diarrhea, are the best approach.

Question 8 A 54-year-old man presents with several weeks of diarrhea and joint pain. His evaluation
yields the diagnosis of Whipple's disease. He receives IV ceftriaxone for 14 days, followed by initiation of
TMP-SMX. He presents to the ER 3 weeks later with fever, rigors, and hypotension. His infectious
workup does not reveal an etiology for his fever. His wife reports his diarrhea and joint pain had
resolved last week and that he tolerated a course of TMP-SMX for a skin infection last year.

What is the most likely cause of these symptoms?

A. Refractory Whipple's disease

B. Sulfa allergy

C. MAC infection

D. Immune reconstitution inflammatory syndrome

Sorry, you have selected the wrong answer.

ANSWER: D. Treatment of Whipple's disease can precipitate a syndrome of fever, rigors, and
hypotension, known as the Jarisch–Herxheimer reaction. Patients may also develop an immune
reconstitution inflammatory syndrome.

Question 9 A 24-year-old man presents for evaluation of fatigue. Based on a rash on his buttocks and a
history of a low-velocity fracture and iron-deficiency anemia, celiac disease is suspected. The patient has
already been on a gluten-free diet for the last 8 months due to its perceived health benefits. Upper and
lower endoscopies do not reveal a bleeding source, and small-bowel biopsies show normal villious
architecture with an increased intraepithelial lymphocyte count.

Which of the following tests would be most useful in excluding celiac disease in this patient?

A. A repeat upper endoscopy with biopsies of both the duodenal bulb and the second part of the
duodenum
B. A serum tTG

C. A serum endomysial antibody

D. A skin patch test for gluten sensitivity

E. HLA testing

Sorry, you have selected the wrong answer.

ANSWER: E. The best test here is the HLA genotype: those with HLA DQ2 and DQ8 are at most risk, and a
negative test has excellent NPV. In this patient, serological testing may be negative due to the
gluten-free diet. Though some studies have suggested a higher yield with biopsies of the duodenal bulb
and the second portion of the duodenum, in this patient repeat biopsies are unlikely to help exclude the
disease (unless he goes back on a gluten-rich diet). Skin patch testing for gluten sensitivity has no role in
the diagnosis of celiac disease.

Question 10 A 36-year-old man with ulcerative colitis has not responded to 5-ASA compounds, nor to
infusions of infliximab. He is commenced on vedolizumab and achieves a clinical response. However, he
is concerned about long-term side effects.

Which of the following is the most common side effect of vedolizumab?

A. Infusion-associated pyrexia

B. Nasopharyngitis

C. Skin rash

D. Upper respiratory infection


E. Pancreatitis

Sorry, you have selected the wrong answer.

ANSWER: B. Nasopharyngitis occurs in up to 13% of patients treated with vedolizumab, an IgG 1


monoclonal antibody. Rash, infusion-associated pyrexia, and upper respiratory tract infections occur in
less than 10%. Pancreatis is a rare side effect.

Question 11 Which of the following statements regarding gut peptides is not correct?

A. Gut peptides are produced in response to the presence of specific types of nutrients in the gut lumen

B. CCK is the prototypical satiety hormone, produced by cells in the duodenum and jejunum in response
to the presence of fat and protein within the gut lumen

C. The peptide ghrelin is predominantly produced by the pancreas in response to fasting and weight loss

D. The actions of GLP-1 include inhibition of gastric emptying, stimulation of insulin release, inhibition of
glucagon release, and inhibition of appetite

E. Ghrelin is the only known circulating appetite stimulant

Well done, you have selected the right answer.

ANSWER: C. Hormones that signal satiety include CCK, PYY and GLP-1. Ghrelin is a 28 amino acid peptide
that increases food intake and promotes weight gain (via the vagus and centrally). The grehlin levels fall
after a meal but rise abruptly prior to eating. Ghrelin is the only known circulating hormone that
increases food intake. Ghrelin is produced mainly by ghrelin cells in the fundus in response to distension.

Question 12 A 45-year-old Caucasian female develops acute pancreatitis and subsequently has four
further episodes of pancreatitis over a period of 1 year. She had a cholecystectomy 10 years ago and her
liver tests are normal during each episode of pancreatitis. She denies any alcohol use. Other metabolic
causes of pancreatitis, such as hypercalcemia and hypertriglycericemia, have been ruled out. She takes a
beta-blocker for hypertension and low-dose aspirin. CT imaging of the abdomen reveals small cystic
lesions (maximal size 0.8 cm) in the head of the pancreas and a normal pancreatic duct. Repeat CT scan
in 6 months reveals the cysts are stable.

What are the diagnosis and the most appropriate next step?

A. IPMN causing pancreatitis; refer for consideration of a Whipple procedure

B. IPMN; perform EUS to evaluate the pancreatic cysts and perform FNA

C. Idiopathic recurrent pancreatitis; consider ERCP and pancreatic stent placement

D. Drug-induced pancreatitis; stop the aspirin

E. Choledocholithiasis causing pancreatitis; perform ERCP and biliary sphincterotomy

Sorry, you have selected the wrong answer.

ANSWER: A. IPMNs are benign tumors of the liver with malignant potential. In this case, the patient has
imaging features consistent with branch-duct IPMN. She has had five episodes of acute pancreatitis and
other etiologies, such as gallstones, alcohol, and metabolic causes, have been ruled out, so IPMN is the
main risk factor for her recurrence. The main indications for resection of IPMN are main pancreatic duct
involvement, large (>3 cm) cyst, intramural nodules, and abnormal cytology. If IPMN is strongly
suspected to be causing the recurrent pancreatitis then the patient should be referred for surgical
evaluation to prevent further pancreatitis.

Question 13 A 65-year-old woman reports a 3-year history of constipation and fecal incontinence. She
strains excessively, requiring 15 minutes or longer to evacuate soft-formed stool from the rectum, and
feels incompletely evacuated after a bowel motion. Adequate dietary fiber supplementation initially
improved her symptoms but the effect has diminished with time. She has tried osmotic laxatives,
enemas, and secretagogues (linaclotide and lubiprostone); these agents improved evacuation but were
discontinued because they caused accidental bowel leakage. She was unaware of this until she felt a wet
sensation in her underwear. Digital rectal exam discloses reduced anal resting tone, a normal squeeze to
voluntary command, and reduced perineal descent during simulated defecation. Anorectal testing
reveals reduced anal resting pressure, a normal squeeze pressure increment, and an abnormal balloon
expulsion test. A CBC and routine chemistry panel (including serum calcium and TSH) are normal. A
colonoscopy was normal 1.5 years ago.

What is the most appropriate next step?

A. Repeat a colonoscopy

B. Perform a defecating proctogram

C. Refer to a surgeon for colectomy

D. Increase dietary fiber content

E. Recommend an evacuation regimen using stimulant suppositories

Sorry, you have selected the wrong answer.

ANSWER: E. This patient describes features of both an evacuation disorder and fecal incontinence. An
evacuation regimen with bisacodyl or glycerol suppositories will aid evacuation without altering fecal
consistency and predisposing to fecal incontinence. If this strategy fails, referral for pelvic-floor
biofeedback therapy may be warranted. A repeat colonoscopy is unlikely to yield further useful
information. While a defecating proctogram may provide information regarding anorectal function,
simple conservative medical strategies should be tried before further investigations are requested.
Additional fiber is unlikely to provide additional benefit but can be trialled. A colostomy is a last
treatment option for patients with severe constipation or fecal incontinence.

Question 14 A 51-year-old woman reports mixed passive and urge fecal incontinence for the last year.
She has accidental bowel leakage of moderate-volume liquid and semi-formed feces twice a week. She
also reports postprandial diarrhea with urgency on a daily basis. She does not drink caffeinated
beverages, use artificial sweeteners, or report symptoms of lactose intolerance, fructose intolerance, or
small-intestinal bacterial overgrowth. She does not take laxatives. As a consequence of her
incontinence, she has withdrawn from social activities. She reports three vaginal deliveries over 25 years
ago. One delivery was complicated by a grade 3 perineal tear and one required forceps to assist delivery.
Her surgical history is pertinent for a hysterectomy 5 years ago and a cholecystectomy for symptomatic
cholelithiasis 2 years ago. Digital rectal exam discloses reduced anal resting tone and reduced squeeze
to voluntary command. Anorectal testing reveals reduced anal resting pressure and a reduced squeeze
pressure increment. A CBC and TSH are normal. A colonoscopy with biopsies was normal 2 months ago.

What is the most appropriate next step?

A. Therapeutic trial with a bile acid-binding agent

B. Assessment of the anal sphincters by endoanal ultrasound

C. Assessment of the anal sphincters by MRI

D. Referral for pelvic-floor biofeedback therapy

E. Referral for sacral nerve stimulation (SNS)

Well done, you have selected the right answer.

ANSWER: A. Fecal incontinence in this patient may be attributable to the combination of semi-formed
stools, urgency, and pelvic-floor weakness. The patient reports symptoms consistent with
postcholecystectomy diarrhea. Though she has risk factors for anal sphincter injury, this is less likely to
be the primary cause of her incontinence. Physical therapy, surgery, bulking agents, and sacral nerve
stimulation should be reserved for those patients who have symptoms despite optimization of bowel
habits. Thus options for anatomically defining her sphincter at this stage would not be the first priority.

Question 15 An 83-year-old female nursing home resident has recurrent UTIs with Escherichia coli. On
the latest occasion, she received ciprofloxacin, and 1 week after stopping it, she developed severe
diarrhea. A stool PCR assay is positive for Clostridium difficile. The patient is treated with vancomycin
(125 mg four times daily), with some improvement. Subsequently, however, the infection recurs with
more severe diarrhea, and she is sent to the emergency department. On physical examination, she
appears disoriented. Temperature is 38.4 °C, pulse rate is 98 bpm, and blood pressure is 90/60 mmHg.
The abdomen is distended. Bowel sounds are scant. Her WBC count is elevated at 32,000 and her
creatinine has doubled from baseline. She is admitted to ICU and commenced on vancomycin 250 mg
orally four times daily. An abdominal X-ray shows a dilated colon.

Which additional measure is most likely to be of benefit?

A. Vancomycin 500 mg IV four times daily

B. Fidaxomicin 100 mg orally twice daily

C. Upper endoscopy with duodenal fecal microbiota transplant

D. Intracolonic vancomycin

E. IV metronidazole 500 mg daily

Sorry, you have selected the wrong answer.

ANSWER: D. This patient has an ileus, and oral antibiotics may not be effective. The addition of
intracolonic vancomycin in this setting may avoid the need for colectomy. IV antibiotics are also usually
added in this scenario, but the correct dose of metronidazole is 500 mg three times daily. IV vancomycin
has no effect as vancomycin is not excreted into the colon when administered this way. While there are
limited case reports of fecal microbiota transplantation in patients with severe colitis, this is unlikely to
work via the upper GI tract in this case due to the ileus. Fidoximicin is also effective when administered
orally, but it has a limited role in a severe C. diff infection complicated by an ileus.

Question 16 A 65-year-old man is diagnosed with ulcerative proctitis. He has three or four bloody bowel
movements daily. His colonoscopy reveals only inflammation in the rectum and distal sigmoid colon. The
patient refuses an enema and will only take a pill.
Which therapeutic option should be recommended?

A. Commence prednisone 20 mg daily

B. Commence azathioprine 2 mg/kg daily

C. Commence 5-ASA 4.8 g in three divided doses

D. Commence controlled-release 5-ASA 2–4 g in three divided doses

E. Commence budesonide 9 mg daily

Sorry, you have selected the wrong answer.

ANSWER: D. This patient is making an unfortunate choice by not considering topical steroid- or
5-ASA-based therapy, which is effective in proctitis. Allowing for this, a controlled-release substance is
likely to achieve better drug delivery in this scenario. Thus, the oral prednisone, azathioprine, and oral
budesonide would not be ideal first choices.

Question 17 A 35-year-old man is diagnosed with FAP after observation of hundreds of adenomatous
polyps during a colonoscopy and positivity on genetic testing. He has read on the Internet that FAP also
puts him at risk of extracolonic malignancies.

Which of the following malignancies would he not be at increased risk of despite his FAP syndrome?

A. Duodenal adenocarcinoma

B. Gastric adenocarcinoma
C. Follicular thyroid cancer

D. CNS tumors

E. Renal cell carcinoma (RCC)

Sorry, you have selected the wrong answer.

ANSWER: E. The lifetime risk of duodenal adenocarcinoma among patients with FAP is 5–10%, and
small-bowel surveillance is recommended. There is also an increased lifetime risk of gastric, thyroid
(both follicular and papillary), and CNS tumors (mainly medulloblastoma), but not RCC.

Question 18 A 67-year-old man undergoes a screening colonoscopy due to a positive family history of
early-onset bowel cancer in a first-degree relative. The colonoscopy identifies four polyps and one
submucosal lesion. All the polyps, but not the submucosal lesion, are removed completely.

Which of the following lesions identified during this colonoscopy has the greatest potential for
neoplastic transformation?

A. A 5 mm tubular adenoma with low-grade dysplasia removed from the cecum

B. A 7 mm tubulovillous adenoma removed from the sigmoid colon

C. A 3 mm hyperplastic polyp removed from the rectum

D. An 8 mm inflammatory pseudopolyp

E. A 10 mm submucosal lesion with positive tenting sign


Sorry, you have selected the wrong answer.

ANSWER: B. The 7 mm polyp with tubulovillous features has the highest risk of malignant
transformation as the most important risk factors are polyp size (larger size means a higher risk of
dysplasia), presence of villous features, and high-grade dysplasia. The 5 mm tubular adenoma with
low-grade dysplasia is the most common form of colonic polyp and is associated with low risk of
malignant transformation. The 3 mm hyperplastic rectal polyp is associated with very low risk of
malignant transformation. The 10 mm submucosal lesion with positive tenting sign indicates a harmless
submucosal lipoma. Inflammatory pseudopolyps are benign.

Question 19 Which of the following should be restricted in the diet of a patient with short-bowel
syndrome who has approximately 50% of their colon remaining, anastomosed to the residual small
intestine?

A. Protein

B. Magnesium

C. Oxalate

D. Calcium

Sorry, you have selected the wrong answer.

ANSWER: C. Oxalate is usually bound to dietary calcium and eliminated in the feces. In patients with
short-bowel syndrome and fat malabsorption, calcium preferentially binds to fatty acids. The oxalate
passes into the colon, where it is absorbed into the bloodstream, and is filtered in the kidney, where it
may lead to hyperoxaluria. It binds to calcium, and calcium oxalate nephrolithiasis may develop. Fat
restriction, oral calcium supplementation, and a low-oxalate diet are useful therapies.

Question 20 Which of the following best prevents dehydration in a patient with short-bowel syndrome?

A. Fat restriction
B. Water

C. Oral rehydration solution

D. Double-sealed ostomy appliance

Sorry, you have selected the wrong answer.

ANSWER: C. The use of oral istonic or slightly hypo-osmolar oral rehydration solutions is critical for
adequate fluid management. Oral rehydration solutions take advantage of the jejunal sodium–glucose
co-transporter and solvent drag to increase intestinal water absorption. Water is hypotonic and
sodium-free, and may result in sodium excretion and worsen dehydration. Fat restriction may decrease
diarrhea slightly in patients with residual colon in continuity with their small bowel.

Question 21 A 25-year-old woman reports that while having a few drinks at a bar with her friends, she
ate from a bowl of nuts that was on the counter. About 20 minutes later, she developed abdominal
cramps and vomiting. She states that she does not feel her alcohol intake was excessive. She has never
had a reaction to nuts in the past, and none of her friends who also ate the nuts had similar symptoms.
She currently feels well, but presents with questions regarding the cause of her symptoms and whether
there is need for future concern.

Which of the following statements is true?

A. The patient can be assured that the reaction she experienced is not related to the ingested nuts, as
the time between the ingestion and the onset of symptoms is too short

B. Since the reaction was mild, even if it was caused by the ingested nuts, future reactions will also be
mild

C. Allergic reactions to nuts typically involve the respiratory and cardiovascular system and rarely involve
the GI tract
D. Allergic reactions to nuts are a disease of the young and generally resolve over time

E. Given the history, detection of serum IgE to peanut or a positive skin-prick test is enough to make the
diagnosis of nut allergy

Sorry, you have selected the wrong answer.

ANSWER: E. Allergen-specific IgE can be detected by serum IgE levels or skin-prick tests. A double-blind
placebo-controlled food challenge (DBPCFC) is the most specific test for food allergy and reliably
distinguishes sensitization from clinical allergy, but it is expensive and labor-intensive. Therefore, if
serum IgE and skin-prick tests are unequivocal and are coupled with a consistent history, a DBPCFC is
not needed to confirm the diagnosis.

IgE-mediated reactions are characterized by acute onset of symptoms, generally occurring within
minutes to hours after ingestion of the trigger food. The severity of a reaction cannot be accurately
predicted by the severity of past reactions. Though reactions following a severe reaction are likely also
to be severe, those following mild reactions can be severe too. IgE reactions to foods typically involve
the skin (urticaria, angioedema, erythema, pruritus), GI tract (vomiting, abdominal pain), and respiratory
tract (persistent cough, hoarse voice, wheeze, stridor, respiratory distress, nasal congestion). Milk, egg,
and peanut account for the vast majority of IgE-mediated allergic reactions in children, while peanuts,
tree nuts, and seafood account for the majority in teens and adults.

Question 22 A 23-year-old woman presents with abdominal bloating, gas, and alternating bowel habits,
which are worse with ingestion of wheat and dairy products. She has a second cousin with celiac
disease. She has undergone celiac serologies and genetic testing at her primary care office. Serum tTG
and endomysial antibody are negative on a normal gluten-containing diet and she is not IgA-deficient,
but she carries the genes for the HLA DQ2 receptor. EGD is performed and a Marsh I lesion is seen on
biopsies.

Which of the following statements is true?

A. The small-bowel biopsy is highly likely to indicate celiac disease


B. The patient should be evaluated for milk allergy using a skin-prick test

C. The positive celiac-associated genetics make it highly likely that this patient will develop celiac disease

D. The patient's intolerance to wheat may be caused by the fermentable and osmotic properties of
wheat, rather than the presence of gluten itself

E. The patient's gluten intolerance is likely to be caused by an IgE-mediated reaction to wheat

Sorry, you have selected the wrong answer.

ANSWER: D. In a randomized trial of patients with irritable bowel symptoms who had biopsies and/or
genetic markers negative for celiac disease but who felt that their symptoms were exacerbated by
ingestion of wheat, symptomatic improvement was seen in those on a low-FODMAP diet, regardless of
the amount of gluten in the diet. Though it is possible that a Marsh I lesion will eventually progress to a
more advanced histologic stage, studies suggest that only about 15% actually do. Other etiologies that
result in this histology include drugs such as aspirin and NSAIDs, H. pylori infection, bacterial infection of
the small bowel, and immune dysregulation (autoimmune disorders). Cow's milk allergy is much more
prevalent in early childhood, with an incidence of 2–6%. Most affected infants acquire tolerance by the
age of 5, and the incidence in adulthood is 0.1–0.5%. When immune reactions do occur in adults, they
are rarely IgE-mediated, so a skin-prick test is not the most effective way of evaluating the relationship.
Though >95% of patients with celiac disease have the HLA DQ2 or DQ8 receptor, the converse is not
true: about 30–40% of Caucasians carry DQ2 or DQ8 but <3% will develop celiac disease. This suggests
that though necessary, DQ2 and DQ8 are not sufficient to cause celiac disease. Gluten intolerance has
been ascribed to an innate immune response to wheat and a non-IgE-mediated immune response to
wheat proteins, dietary FODMAPs, etc., and is likely a heterogeneous process. However, before
entertaining a diagnosis of gluten intolerance, both celiac disease and an IgE-mediated wheat allergy
must be ruled out.

Question 23 A 75-year-old male presents with a 6-month history of progressive abdominal pain and
weight loss of 40 lb. He seeks medical attention because he has developed skin discoloration, pale
stools, and dark urine. He has a 40-pack-year history of smoking and drinks two glasses of wine every
night. Exam reveals a cachectic-appearing male with conjunctival icterus. Abdomen is soft,
non-distended, and non-tender. Blood tests show normal hemoglobin and normal WBC count. His liver
tests are notable for a predominantly direct hyperbilirubinemia (12.5 mg/dL) and elevated liver enzymes
(AST 210 IU/L, ALT 140 IU/L, alkaline phosphatase 330 IU/L). Ultrasound abdomen reveals a dilated
common bile duct (11 mm). Distal common bile duct is poorly visualized due to overlying bowel gas.
What is the most appropriate next step?

A. ERCP with biopsy and stenting to relieve biliary obstruction

B. EUS with evaluation of the biliary tree and pancreas

C. CT abdomen with IV contrast

D. EGD with small-bowel biopsy

E. MRCP with MRI

Sorry, you have selected the wrong answer.

ANSWER: C. CT and the collection of images during the arterial and portal venous phase (“pancreas
protocol”) leads to accurate diagnosis and staging in about 90% of cases of pancreatic cancer and has
been extensively validated. Vascular invasion involving the superior mesenteric artery, celiac axis,
superior mesenteric vein, and splenic vein can be diagnosed on CT scan. EUS is more sensitive for
tumors <2 cm and is useful for staging, but this patient likely has an advanced tumor, given the presence
of obstructive jaundice. MRI with MRCP can be used in patients with contrast allergy, but they are not
superior to CT. Stent placement and ERCP are generally reserved for palliation of biliary obstruction in
patients who are not surgical candidates.

Question 24 A 70-year-old male is admitted to the hospital for evaluation of acute severe abdominal
pain radiating to his back. His lab tests show a lipase of 4500 IU/mL, giving him a clinical diagnosis of
acute pancreatitis. His liver tests are normal and an ultrasound of the liver shows normal-caliber
common bile duct with no evidence of cholelithiasis. He denies alcohol use. He was recently diagnosed
with diabetes mellitus. His serum calcium and triglycerides are normal. His medications include low-dose
aspirin, metformin for diabetes, and simvastatin for hyperlipidemia. He recovers from his acute episode
and is discharged home. He is seen in the outpatient setting 3 months after discharge and is
asymptomatic.
What is the most appropriate next step, and why?

A. No further testing is needed: this is likely idiopathic pancreatitis

B. EUS, to evaluate for pancreatic cancer

C. Cholecystectomy, to prevent further episodes of gallstone pancreatitis

D. CT abdomen, to ensure resolution of pancreatitis

E. ERCP, to place prophylactic pancreatic stents

Sorry, you have selected the wrong answer.

ANSWER: B. Idiopathic pancreatitis in the elderly raises suspicion for an obstructing pancreatic tumor.
Abdominal imaging with either CT scanning or EUS is recommended 3–4 months after resolution of
pancreatitis to rule out pancreatic cancer or IPMN. EUS can also help assess for biliary microlithiasis and
evidence of undiagnosed chronic pancreatitis.

The recent onset of diabetes in this elderly patient also raises concern for pancreatic adenocarcinoma.
Approximately 5% of patients with pancreatic adenocarcinoma present with acute pancreatitis as their
initial symptom. A cholecystectomy is not recommended in this patient as he does not have evidence of
cholelithiasis or choledocholithiasis. ERCP is no longer performed as a diagnostic test and is reserved for
therapeutic indications alone.

Question 25 A 72-year-old Caucasian man is diagnosed with mid-jejunal carcinoid tumor when he
presents for small bowel obstruction and undergoes exploratory laparotomy with resection and primary
anastomosis. Further evaluation for metastasis is negative.

Which of the following statements is true regarding the follow-up of this patient?
A. The most sensitive screening method for any recurrence is an octreotide scan

B. Long-acting somatostatin analogues can significantly improve survival in patients with metastatic
mid-gut NET

C. Serum markers are useful in making a diagnosis of NETs but should not be used to follow recurrence
in a patient

D. Surgical resection has no role in patients diagnosed with NETs once metastasis has occurred

Sorry, you have selected the wrong answer.

ANSWER: D. Several studies have shown that long-acting somatostatin analogues can significantly
lengthen progression-free survival in patients with functionally active and inactive metastatic mid-gut
NET. Octreotide scan is indicated as the first staging procedure and is one of the most sensitive
single-screening methods for extrahepatic disease, but it is not first-line to screen for hepatic
involvement. The various peptides and biogenic amines secreted from the neuroendocrine cells are
tumor-specific and may serve as markers for the diagnosis and follow-up of treatment. Even in patients
with metastasis, resection of the primary tumor, if feasible, has been shown to offer a survival
advantage.

Question 26 Which of the following statements is NOT true regarding secretin?

A. Secretin aids in MRI of the pancreas and ductal cannulation during ERCP

B. The secretin stimulation test is a sensitive and specific diagnostic test for Zollinger-Ellison syndrome

C. The primary physiologic action of secretin is inhibition of pancreatic fluid and bicarbonate secretion

D. A low secretin level is seen in untreated celiac disease, which contributes to the maldigestion
associated with celiac disease

E. Secretin turns off gastric acid seretion.

Sorry, you have selected the wrong answer.

ANSWER: C. Secretin stimulates pancreatic bicarbonate and fluid. It is released by the duodenum in
response to acid. The pH change in the duodenum switches off secretin release. Secretin also inhibits
gastric acid secretion. In Zollinger- Ellison syndrome (ZE), fasting hypergastrinemia plus exclusion of
achlorhydria is required for diagnosis. A false positive secretrin test can occur in the setting of
achlorhydria and should not be relied on in this situation for diagnosis of ZE.

Question 27 A 46-year-old woman is evaluated for bilateral knee pain. The pain has been present for 6
years, and worsens after physical activity. X-rays show bilateral degenerative changes consistent with
osteoarthritis. She has a BMI of 48 kg/m2 and hypertension, dyslipidemia, and T2DM with recent
glycosylated hemoglobin on 9.7% despite recent attempts at lifestyle changes and a low-calorie diet. She
has heartburn once a month, but no dysphagia or regurgitation. Her current medications include
metformin, metoprolol, lisinopril, atorvastatin, and acetaminophen.

What is the most appropriate next step?

A. Intensify lifestyle and medical management

B. Refer for fundoplication

C. Refer for gastric bypass (RYGB)

D. Prescribe orlistat for 6 months

E. Proceed with bilateral knee replacement to facilitate the patient's ability to exercise and lose weight
naturally
Well done, you have selected the right answer.

ANSWER: C. In a patient with severe obesity (class III), RYGB is the most appropriate next step. The
patient's attempts at lifestyle modification have failed and bilateral knee osteoarthritis lessens the
chances of successful natural weight loss. RYGB results in an average excess body weight loss of 67.5% at
1 year, with remission of diabetes in 90%, of hypertension in 80%, of obstructive sleep apnea in 95%,
and of dyslipidemia in 70%. Adjustable gastric banding is much less effective and has been associated
with high rates of adverse events. Orlistat promotes modest weight loss on the order of 2–3 kg, which is
unlikely to significantly impact this patient's comorbidities. The patient may require knee surgery, but
she is currently a high anesthetic risk. Thus, her severe obesity must be addressed. Her reflux is mild and
requires no treatment or symptomatic control and even if it was a fundoplication given her BMI would
be contraindicated.

Question 28 A 33-year-old woman presents to the emergency department with nausea that has
progressively worsened over the past few weeks. She mentions associated numbness and tingling in her
fingers and toes. Over the last few days, she has had difficulty concentrating, and she is concerned
about some mild changes in vision. Her past health is significant for cholecystectomy 8 years ago and
roux en Y gastric bypass (RYGB) 18 months ago for medically complicated obesity. Her blood sugar is 145
mg/dL.

What treatment should be started while further evaluation is undertaken?

A. Administer IV glucose

B. Administer IV octreotide

C. Administer IV thiamine

D. Administer normal saline

E. Administer insulin bolus followed by insulin infusion


Sorry, you have selected the wrong answer.

ANSWER: C. This patient has developed thiamine deficiency. Asymptomatic thiamine deficiency can be
seen in 20% of post-bariatric surgery patients within 1 year. Non-specific symptoms such as nausea and
mild neurological symptoms are usually present in more advanced deficiency. There are cases of severe
thiamine deficiency presenting with Wernicke–Korsakoff syndrome following bariatric surgery, but this
is rare. There should be a high index of suspicion for thiamine deficiency in a patient with subtle
neurological symptoms after RYGB. Glucose, octreotide, insulin, or normal saline administration would
likely not be beneficial for this patient at this stage.

Question 29 A 46-year-old man with known chronic intestinal pseudo obstruction (CIPO) presents to the
ER with worsening symptoms for the third time in 3 months. He is vomiting, is dehydrated, and has
noticed reduced urine output. His CIPO has been symptomatically managed with small, frequent meals
and metoclopramide. Empiric antibiotics for possible SIBO have been ineffective. In the last 3 months,
he has lost 7 kg of body weight. On examination, he is tachycardic and appears unwell, with diffuse
abdominal tenderness and no peritoneal signs. An abdominal CT shows severe dilation of the small
bowel and colon, but not mechanical obstruction.

What is the most appropriate next step?

A. Admit for IV rehydration and commence TPN

B. Admit to hospital for hospice management, given his nutritional failure and incurable condition

C. Admit for IV rehydration and refer to an expert center for consideration of intestinal transplantation

D. Admit for IV rehydration and commence enteral feeding via gastrostomy or jejunostomy

E. Admit for IV rehydration and consider surgical resection of affected areas of the intestine

Well done, you have selected the right answer.


ANSWER: D. This patient has progressive CIPO and is no longer tolerating oral nutrition. Despite the
inability to take oral nutrition, many CIPO patients can be managed with slow enteral feeding via a
gastrostomy or jejunostomy tube. In some patients, a significant venting effect from the tube adds to
the therapeutic benefit. Complete avoidance of enteral feeding via TPN is premature. Despite the
incurable nature of CIPO, this patient is inappropriate for hospice, as several options for enteral and/or
parenteral nutrition still remain. Intestinal transplantation should only be considered in patients who
have contraindications to or fail TPN. Surgical resection of affected areas is not recommended as no
therapeutic benefit is noted in most patients and the loss of intestinal length may exacerbate nutritional
deficiencies.

Question 30 An 89-year-old woman is 48 hours status post repair of a femoral neck fracture. She is
vomiting, has a distended abdomen, and is obstipated. Physical examination reveals a normal blood
pressure and a heart rate of 48 bpm. She has a mildly tender abdomen without focal peritonism. Scant
bowel sounds are heard. An abdominal X-ray shows a few fluid levels, but gas is present in the rectum.
An abdominal CT does not show any dilation of the small bowel or colon. It is decided that a nasogastric
tube should be placed, narcotic analgesia should be minimized, and hydration and electrolytes should be
monitored.

In addition to the aforementioned measures, which of the following management strategies is optimal?

A. Obtain surgical review of consideration of exploratory laparotomy

B. Transfer the patient to a monitored bed and administer neostigmine

C. Closely monitor the patient and re-examine the abdomen every 6 hours

D. Place a rectal tube

E. Perform a colonoscopy to endoscopically decompress the patient

Sorry, you have selected the wrong answer.


ANSWER: C. This patient most likely has a postoperative ileus and is likely to spontaneously improve
with supportive care. The presence of gas in the rectum makes complete small-bowel obstruction less
likely. Surgery is only indicated in cases of complete obstruction with evidence of complications. The
absence of colonic dilation makes acute colonic pseudoobstruction less likely, and hence neostigmine is
unlikely to help. Furthermore, the patient's bradycardia is a relative contraindication to neostigmine.
Endoscopic decompression with placement of a colonic decompression tube can be effective in cases of
acute colonic pseudoobstruction when neostigmine is contraindicated or has failed failed.

Question 31 A 53-year-old man in the surgical ward presents with severe uncontrolled abdominal pain
radiating to the chest, new left shoulder pain, and a sense of doom 3 days after a laparoscopic RYGB
surgery. He tolerated a diet and pain medications per oral this morning. His heart rate is 140 bpm and
his labs show a WBC count of 16,000.

What is the most likely diagnosis?

A. Partial small-bowel obstruction

B. Pulmonary embolism

C. Anastomotic leak

D. Postoperative and narcotic-induced ileus

E. Myocardial infarction

Sorry, you have selected the wrong answer.

ANSWER: C. Anastomotic leaks are the most common early complications after laparoscopic RYGB
surgery, occurring less than 5% of the time. Most large series report leak rates of 0.5–2.0%. The most
common site is at the gastrojejunostomy, followed by the jejunojejunostomy. Persistent tachycardia,
tachypnea, or hypoxia should alert the clinician to look for an anastomotic leak even when other
symptoms or laboratory abnormalities are absent. In this clinical scenario, this is the most likely
condition, as partial small-bowel obstruction and postoperative ileus usually produce nausea and
vomiting. Pulmonary embolism can cause tachycardia and hypoxia with chest pain, but does not cause
severe abdominal pain.

Question 32 A 76-year-old woman presents with acute-onset vomiting, abdominal pain, and obstipation.
Her past surgical history is significant for being status post transabdominal hysterectomy and
laparoscopic appendectomy. On examination, she is afebrile and tachycardic, and has diffuse abdominal
tenderness without focal peritonism. A plain abdominal film reveals multiple fluid levels and dilated
small bowel.

After commencing fluid resuscitation, what is the most appropriate next step?

A. Abdominal CT

B. CT enterography

C. MRI enteroclysis

D. Small-bowel scintigraphy

E. Laparotomy without further imaging

Well done, you have selected the right answer.

ANSWER: A. This patient most likely has an adhesive small-bowel obstruction. A CT has good sensitivity
and specificity for detecting the cause and identifies potential complications and is thus the most
appropriate next step. A CT enterography also has high sensitivity for small-bowel obstruction, but is
used when routine CT fails to identity the cause and is relatively contraindicated given the concern for
complete obstruction. An MRI enteroclysis is also contraindicated in this patient, as aggressive inflation
of the bowel with contrast is required. A small-bowel scintigraphy would not be useful. A laparotomy is
not indicated at this time, as the patient has no definitive signs of peritonism and is afebrile. A large
proportion of patients with small-bowel obstruction will resolve with conservative management, and
surgery is reserved for patients who either do not resolve or have evidence of a complication such as
peritonism or strangulation.
Question 33 Which of the following is an appropriate indication for intestinal transplantation?

A. Advanced hepatic fibrosis

B. Mesenteric vein thrombosis

C. Presence of a jejunostomy

D. Folate deficiency

Well done, you have selected the right answer.

ANSWER: A. Medicare criteria for intestinal transplantation, largely adopted by other third-party payers,
include the debatable indications of recurrent bacterial sepsis or a single fungal infection, as well as loss
of three central veins. However, the primary indication is liver failure. A reasonable argument can be
made that patients should be transplanted prior to development of hepatic failure, at a time when an
isolated intestinal transplant can be performed. Isolated liver transplantation in patients with intestinal
failure is associated with poor outcome. Given the current long-term survivals of patients with intestinal
failure and those following transplant, intestinal transplantation is not currently appropriate for stable
patients with short-bowel syndrome.

Question 34 Which of the following drugs acts on the intestinal chloride channels to relieve
constipation?

A. Prucalopride

B. Polyethylene glycol

C. Lubiprostone

D. Linaclotide
E. Tegaserod

Sorry, you have selected the wrong answer.

ANSWER: C. Lubiprostone acts on the type 2 chloride channels (CIC-2). Linaclotide is a guanylate GC-C
agonist, prucalopride and tegaserod act on the 5HT4 receptor, and polyethylene glycol is an osmotic
laxative.

Question 35 A 62-year-old man presents with a 1-day history of maroon stools with clots. His past
medical history is notable for hyperlipidemia, coronary artery disease, and depression. He had a
screening colonoscopy 1 year ago, which was remarkable for diverticulosis and internal hemorrhoids. He
takes simvastatin and aspirin (81 mg). On presentation, his blood pressure is 100/65 and his heart rate is
102 bpm. Abdominal exam is benign. His initial CBC and metabolic panel are notable for a hematocrit of
27%. After 2 L of normal saline, he is no longer tachycardic or hypotensive.

What is the most appropriate next step?

A. Continue monitoring without plans for intervention unless the patient destabilizes

B. Refer the patient for a tagged RBC scan

C. Plan for a colonoscopy the next day, whenever an endoscopy slot is available

D. Begin bowel preparation in anticipation of colonoscopy

E. Perform a CT angiography
Sorry, you have selected the wrong answer.

ANSWER: D. Though this patient has had a fairly recent colonoscopy, a repeat exam would be indicated
in the setting of acute bleeding to identify and potentially treat the bleeding source and to exclude
ominous lesions. Tagged RBC scanning (radionuclide scintigraphy) is sensitive to low rates of bleeding.
However, it fails to accurately localize the source of bleeding in up to 45% of patients and it has no
therapeutic capabilities. Therefore, tagged RBC scanning is not the optimal test for patients with
hematochezia presumed to be from the colon. Radiographic diagnosis of GI bleed with tagged RBC
scanning, angiography, or CT angiography requires active bleeding at the time of the exam, so these
tests are only appropriate in patients with significant, ongoing bleeding. Colonoscopy is the preferred
initial strategy for the majority of patients with lower GI bleed. Though results from randomized trials
are limited, the studies suggests that colonoscopy is particularly useful when performed within 24 hours
of presentation, and that it yields more definitive diagnoses and may improve other outcomes (e.g.,
rebleeding, need for surgery, length of hospital stay) when compared to the standard of care (elective
colonoscopy in stable patients and tagged RBC scan followed by angiography in patients with ongoing
bleeding). Definitive bleeding sources are rarely identified during elective or delayed colonoscopy for
lower GI bleed.

Question 36 A 75-year-old man presents with iron-deficiency anemia but no signs of overt bleeding. His
past health is significant for moderate aortic stenosis, hypertension, and hypercholesterolemia. He has
undergone two negative bidirectional endoscopies (EGD with small-bowel biopsies and colonoscopies)
in the last 12 months. He has persistent anemia despite taking adequate oral iron supplementation.

What is the most appropriate next step?

A. Capsule endoscopy

B. Repeat EGD and colonoscopy

C. Balloon-assisted enteroscopy

D. Continued iron supplementation


E. Intraoperative enteroscopy

Well done, you have selected the right answer.

ANSWER: A. Despite repeating endoscopies, the patient continues to have medically refractory
iron-deficiency anemia. At this point, a small-bowel evaluation should be performed, and the least
invasive method is capsule endoscopy. A balloon-assisted enteroscopy should be reserved for patients in
whom a diagnostic or therapeutic target has been identified on capsule endoscopy or other imaging.
Furthermore, a capsule endoscopy may help determine the ideal direction of enteroscopy (i.e.,
antegrade vs. retrograde). Continued iron supplementation is inappropriate, and in a patient with aortic
stenosis anemia may lead to morbidity and even mortality. Intraoperative enterscopy is invasive, and
inappropriate for initial small bowel evaluation.

Question 37 A 65-year-old man has refractory iron-deficiency anemia and heme positive stools. An EGD
and colonoscopy do not reveal a bleeding source. A capsule endoscopy is performed and angiectasias
are seen in the proximal and mid small bowel.

What is the most appropriate next step?

A. Push enteroscopy

B. Retrograde device-assisted enteroscopy

C. Anterograde device-assisted enteroscopy

D. CT enterography

E. MRI enterography
Sorry, you have selected the wrong answer.

ANSWER: C. The patient needs endoscopic treatment of the small-bowel angioectasias. Since the
angiectasias are located within the proximal 75% of the small intestine, the best route is an anterograde
device-assisted enteroscopy. A push enteroscopy may not be able to sufficiently intubate the mid small
bowel. Whilst the CT or MRI enterography might be diagnostic this would not add to this current clinical
scenario.

Question 38 A 26-year-old woman is referred to the GI office with diarrhea, urgency, frequency, and
occasional bright red blood per rectum of 6 weeks' duration. She reports no weight loss. She had
bloating and fullness when she was in college. She recently traveled to South East Asia for a backpacking
vacation. Laboratory testing, including a CBC, electrolyte panel, and LFTs, is unremarkable. Stool studies,
including extended testing for ova, cyst, and parasites, are negative. The patient is not willing to undergo
colonoscopy.

Which of the following non-invasive tests is the most sensitive in screening for IBD in a patient such as
this?

A. ASCA and ANCA

B. ASCA and anti-CBIR1

C. ASCA and anti-OmpC

D. NOD-2 SNP testing

E. Fecal calprotectin

Sorry, you have selected the wrong answer.

ANSWER: E. A meta-analysis of 13 studies of both adults and children with chronic diarrhea and
suspected IBD who underwent endoscopy estimated the sensitivity of fecal calprotectin to be 93% (95%
CI: 0.85–0.97) and the pooled specificity to be 96% (95% CI: 0.79–0.99). These results probably do not
translate to general practice, where the pre-test probability will be lower.
ASCA antibodies are found in 40–80% of patients with Crohn's disease, while P-ANCA is positive in
60–80% of adults with ulcerative colitis, making these the next most sensitive – albeit not the most
specific – serological markers for IBD.

Anti-CBIR1 helps differentiate colonic Crohn's from ulcerative colitis, but it is not very sensitive as it is
only found in approximately 50% of patients with Crohn's disease. The anti-OmpC antibody is less
sensitive than ASCA and P-ANCA, but can increase the sensitivity of either marker in combination.

NOD-2 mutations have been found in 42% of children with Crohn's disease.

Fecal calprotectin is elevated in both infectous and inflammatory bowel conditions.

Question 39 A 42-year-old woman who has acute pancreatitis is evaluated for chronic abdominal pain.
She had her first episode of pancreatitis at age 25, and has had multiple hospitalizations since then. She
is quite debilitated by pain and is unable to work and perform her activities of daily living. She has read
about the risk of cancer of the pancreas and is considering a total pancreatectomy with islet
autotransplantation (TPIAT). She undergoes genetic testing to assist in the decision-making process.

During the episodes of intense abdominal pain, laboratory studies show amylase levels more than three
times the upper limit of normal. A recent CT of the abdomen reveals evidence of chronic pancreatitis, an
irregular pancreatic duct, and extensive calcifications within the pancreas. She also has
insulin-dependent T2DM. Her current medications are hydromorphone hydrochloride pancreatic
enzyme supplements (70,000–80,000 USP units of lipase) with meals. She does not smoke cigarettes or
drink alcoholic beverages. Her family history is remarkable for an uncle who was found to have chronic
pancreatitis at age 35.

Which of the following gene mutations is this patient most likely to have?

A. CFTR
B. CTRC

C. PRSS1

D. SPINK1

E. CLDN2

Sorry, you have selected the wrong answer.

ANSWER: C. Mutations in PRSS1 are the most common cause of hereditary pancreatitis, and are present
in up to 80% of patients with hereditary pancreatitis. CFTR mutations are diverse and usually confer low
risk of pancreatitis, while CTRC mutations confer moderate risk. SPINK1 mutations are seen in 1 in 50 in
the general population, but less than 1% of carriers develop chronic pancreatitis; in studies of patients
with chronic pancreatitis, this mutation is encountered in 16–23%. CLDN2 is a risk factor for chronic
pancreatitis among heavy users of alcohol only.

Question 40 Which of the following statements is not true regarding IgA?

A. IgA is the most abundant antibody in the body

B. Secretory IgA's main function is to inhibit the adhesions of viruses and bacteria to epithelium

C. Breast-milk IgA provides passive immunity against pathogens in newborns

D. IgA is produced by goblet cells

E. None of the above


Sorry, you have selected the wrong answer.

ANSWER: D. Goblet cells secrete mucus. Secretory IgA of the intestinal lumen is produced by mature
plasma cells that are derived from mucosal B cells.

Question 41 Which stool electrolyte pattern is most consistent with osmotic diarrhea?

A. Sodium 153 meq/L, potassium 87 meq/L, chloride 160 meq/L, bicarbonate 23 meq/L, pH 8.10,
osmolality 450 mosm/kg

B. Sodium 145 meq/L, potassium 10 meq/L, chloride 100 meq/L, bicarbonate 25 meq/L, pH 7.00,
osmolality 300 mosm/kg

C. Sodium 110 meq/L, potassium 30 meq/L, chloride 90 meq/L, bicarbonate 10 meq/L, pH 6.90,
osmolality 280 mosm/kg

D. Sodium 50 meq/L, potassium 50 meq/L, chloride 50 meq/L, bicarbonate 25 meq/L, pH 7.00,


osmolality 200 mosm/kg

E. Sodium 20 meq/L, potassium 10 meq/L, chloride 15 meq/L, bicarbonate 10 meq/L, pH 7.50, osmolality
300 mosm/kg

Sorry, you have selected the wrong answer.

ANSWER: E. Chronic diarrhea may be from inflammatory diseases, fatty diarrhea (malabsorption), or if
watery may be osmotic or secretory. Hence you need to calculate the stool osmotic gap: 290 − 2 × [Na+]
+ [K+]. In secretory diarrhea there is no osmotic gap while in osmotic diarrhea there is a gap (from
unmeasured osmotically active molecules) of >100m osm/kg. Do not use the measured stool osmolality
to calculate the gap.
In E: 290 − [(2 × 20) + (2 × 10)] = 230 mosm/L. This is osmotic diarrhea.

In D: note the osmolality of stool as measured is well below 290, indicating the addition of water or
hypotonic urine.

Question 41 Which stool electrolyte pattern is most consistent with osmotic diarrhea?

A. Sodium 153 meq/L, potassium 87 meq/L, chloride 160 meq/L, bicarbonate 23 meq/L, pH 8.10,
osmolality 450 mosm/kg

B. Sodium 145 meq/L, potassium 10 meq/L, chloride 100 meq/L, bicarbonate 25 meq/L, pH 7.00,
osmolality 300 mosm/kg

C. Sodium 110 meq/L, potassium 30 meq/L, chloride 90 meq/L, bicarbonate 10 meq/L, pH 6.90,
osmolality 280 mosm/kg

D. Sodium 50 meq/L, potassium 50 meq/L, chloride 50 meq/L, bicarbonate 25 meq/L, pH 7.00,


osmolality 200 mosm/kg

E. Sodium 20 meq/L, potassium 10 meq/L, chloride 15 meq/L, bicarbonate 10 meq/L, pH 7.50, osmolality
300 mosm/kg

Sorry, you have selected the wrong answer.

ANSWER: E. Chronic diarrhea may be from inflammatory diseases, fatty diarrhea (malabsorption), or if
watery may be osmotic or secretory. Hence you need to calculate the stool osmotic gap: 290 − 2 × [Na+]
+ [K+]. In secretory diarrhea there is no osmotic gap while in osmotic diarrhea there is a gap (from
unmeasured osmotically active molecules) of >100m osm/kg. Do not use the measured stool osmolality
to calculate the gap.

In E: 290 − [(2 × 20) + (2 × 10)] = 230 mosm/L. This is osmotic diarrhea.


In D: note the osmolality of stool as measured is well below 290, indicating the addition of water or
hypotonic urine.

Question 2 A 79-year-old man is evaluated for epigastric pain, loss of appetite, and early satiety. Upper
endoscopy reveals a submucosal mass in the gastric antrum. EUS shows a 4 × 6 cm mass in the mid
stomach arising from the second layer (mucosa/lamina propria).

On the basis of the endosonographic appearance, which tumor is most likely?

A. Carcinoid tumor
B. Lipoma

C. Pancreatic rest

D. GIST

E. Duplication cyst

Question 42 A 44-year-old woman is referred for evaluation of chronic constipation that is not
responsive to laxatives and a high-fiber diet. Her major complaints are excessive straining during
evacuation, a sense of incomplete evacuation, and a feeling that something is obstructing the anal canal.
Anorectal studies demonstrate inappropriate contraction of the external anal sphincter during simulated
evacuation.

What is the most appropriate next step?

A. Botulinum toxin injection into the puborectalis muscle bilaterally

B. Botulinum toxin into the external anal sphincter

C. Biofeedback therapy

D. Lateral internal anal sphincter myotomy

E. Kegel exercises

Sorry, you have selected the wrong answer.

ANSWER: C. There are at least four randomized trials which support the effectiveness of biofeedback for
dyssynergic defecation. This may be done with manometry or using EMG techniques. In contrast, an
open-label uncontrolled study suggests that botulinum toxin injections into the puborectalis muscle
using EUS guidance are effective. There are no studies of Botox injections into the external anal
sphincter. Lateral sphincter myotomy is very effective for chronic anal fissures but not for dyssynergic
defecation. Kegel exercises may be helpful in fecal incontinence but will not help pelvic-floor
dyssynergia.

Sorry, you have selected the wrong answer.

ANSWER: A. Carcinoid tumors arise from the second layer (mucosa/lamina propria) and can cause all of
these symptoms. Lipomas are usually asymptomatic, and arise from the third layer (submucosa).
Pancreatic rest usually arises from the third or fourth layer (muscularis propria). Most GISTs arise from
the fourth layer. Duplication cysts are usually asymptomatic and arise from the third layer, or can even
be extrinsic to the GI wall.

Question 43 A 50-year-old woman reports fecal incontinence. Her symptoms began 15 years ago,
approximately 5 years after a fourth-degree perineal tear acquired during a vaginal delivery. At that
time, surgical repair of her external anal sphincter resolved her symptoms. However, the symptoms
have recurred and worsened over the past 3 years. She reports staining her undergarments with a small
amount of stool on several days every week. She describes mostly loose stools with urgency and points
to Bristol 6 on the stool chart. Digital rectal examination demonstrates reduced anal resting tone and no
external sphincter contraction or puborectalis contraction when asked to ‘squeuze’ Endoanal ultrasound
reveals an anterior tear involving 50% of the circumference of the internal and external anal sphincters.

What is the most appropriate next step?

A. Take a careful dietary history regarding the consumption of caffeinated products and sugars (e.g.,
fructose, sorbitol); consider an empiric trial eliminating these products from her diet

B. Refer for pelvic-floor biofeedback therapy

C. Undertake surgical repair of the anal sphincter defects


D. Implant an artificial anal sphincter

E. Refer for diverting colostomy

Well done, you have selected the right answer.

ANSWER: A. Fecal incontinence in this patient may be attributable to the combination of loose stools
and pelvic-floor weakness. Increasing stool consistency and decreasing frequency will often improve
continence. Pelvic-floor biofeedback therapy is not a first-line therapy; it should be reserved for those
patients who have not responded to simple conservative management. After surgical repair of anal
sphincter defects, continence improves in ~80% of patients in the short term but deteriorates over time;
<50% of patients benefit from the operation in the long term. An artificial anal sphincter is associated
with considerable morbidity, particularly wound infections, often requiring device explantation. A
colostomy is a last resort for patients with severe fecal incontinence.

Question 44 A 19-year-old college student presents to the emergency room for evaluation of acute
abdominal pain. He states that he awoke in the middle of the night with an intense, 9-out-of-10
mid-epigastric pain, which moved toward his right lower quadrant over the course of the last hour. With
further questioning, the patient reveals he was in overall good health for most of the day until the pain
began. His review of systems is only significant for a temperature of 101.4 °F. His past medical history is
insignificant, and he has never had any surgeries. The patient does not drink alcohol, smoke cigarettes,
or use illicit drugs, and is currently not sexually active.

If positive, which of the following physical exam findings would suggest the pain is arising from the
abdominal wall?

A. Grey Turner sign

B. McBurney's point tenderness

C. Psoas sign
D. Rosving sign

E. Carnett's test

Sorry, you have selected the wrong answer.

ANSWER: E. Grey Turner's sign is the blue to purple discoloration around a patient's flanks suggestive of
hemorrhagic pancreatitis. The rest of the physical exam findings, listed as B, C, and D, can suggest a
diagnosis of appendicitis. McBurney's point tenderness is discomfort upon palpation at the point
one-third of the way from the superior iliac crest and the umbilicus. A positive psoas sign occurs when
the patient is lying supine, raises his right leg, and feels a painful sensation when the practitioner pushes
the leg down to the ground. A positive Rosving sign is present when the practitioner applies pressure on
the left lower quadrant and the patient feels pain in the right lower quadrant. Tensing of the abdominal
wall muscles causing increased tenderness on palpation suggests abdominal wall tenderness (abdominal
wall pain – Carnett's test-positive).

Question 45 A 56-year-old woman presents for evaluation of increasing constipation. Overall, she
reports feeling in good health, aside from chronic back pain. She reports no allergies, and is currently
taking synthroid, atorvastatin, and thiazide diuretic, in addition to the hydrocodone/acetaminophen
combination pill she has been taking for the last few months for her chronic back pain. A colonoscopy 24
months ago was negative. Recent blood testing showed a normal TSH and calcium. On further
questioning, the patient states that her diet is high in fruit and vegetables. She was a pack-per-day
smoker for 10 years, but quit at the age of 35. She does not drink or do illicit drugs. She denies any
fevers, aches, chills, blood in the stool, dark stools, anorectal pain on defecation, weight changes, or
fatigue. Vital signs include blood pressure of 134/82 mmHg, heart rate 88 bpm, respiratory rate 18
breaths per minute, and temperature 98.4 °F. On cardiovascular exam, there is a physiologic S1 and S2,
with no murmurs rubs or gallops at regular rate and rhythm. Pulmonary exam shows lungs are clear to
auscultation bilaterally in all audible sections. The abdominal examination reveals a normal-appearing
abdomen, normoactive sounds on auscultation, and no pain with palpation in any quadrant. Both liver
and spleen are not enlarged on examination. Digital rectal exam finds external hemorrhoids but no
fissures or other masses. A prominent puborectalis impression is noted on bear-down during the rectal
examination.

What is the most appropriate next step?


A. Refer for an anorectal manometry

B. Recommend cessation of the opiate medication

C. Refer for pelvic floor biofeedback therapy

D. Commence a osmotic laxative

E. Prescribe linaclotide

Sorry, you have selected the wrong answer.

ANSWER: B. This patient presents with constipation with some features of pelvic floor dysfunction. A
reasonable initial step would be to stop opiate medications, which contribute to constipation. If
opiate-free living is not achievable, a trial of an osmotic laxative is reasonable. Anorectal manometry is a
useful test for pelvic floor dysfunction, but first any aggravating factors must be stopped and a simple
laxative regime trialed. Similarly, referral for biofeedback therapy should only be considered after a
positive test (eg. anorectal manometry, defecography).

Question 46 A 31-year-old woman is gravida 1, para 0, 28/40 and presents with constipation that has
persisted despite adequate water and fiber intake. Her TSH and calcium were within normal limits on
pre-natal screening bloodwork. It is decided to add a laxative.

Which of the following suggested therapeutic options is contraindicated in pregnancy?

A. Polyethylene glycol

B. Senokot

C. Castor oil
D. Milk of magnesia

E. Lactulose

Sorry, you have selected the wrong answer.

ANSWER: C. Castor oil is a category X therapy, as it has been associated with uterine rupture. Though
the safety of many laxatives has not been studied in pregnancy, osmotic laxatives and senna are
generally considered safe.

Question 47 A 22-year-old woman with ileocecal Crohn's disease is planning future pregnancy. She is
currently maintained on azathioprine in combination with infliximab. Her last colonoscopy showed
minimally active disease involving the ileocecal valve.

Which of the following is correct regarding IBD and pregnancy?

A. Due to pregnancy-induced immunotolerance, disease activity is generally decreased in pregnancy

B. With few exceptions, medications can be safely continued during pregnancy

C. Most women should deliver by Cesarean section to avoid trauma to the anal sphincters

D. Men should discontinue azathioprine for 3 months before fathering children

E. Women with a history of active disease during pregnancy should be followed by perinatology during
all subsequent pregnancies
Sorry, you have selected the wrong answer.

ANSWER: B. With the exception of methotrexate and thalidomide, medications used to treat IBD should
be continued during pregnancy to control inflammation and maintain maternal health. Though the
thiopurines are a category D medication, evidence compiled over the last few decades suggest that they
are low risk for adverse fetal outcomes. Methotrexate is contraindicated in pregnancy (abortions,
skeletal abnormalities). Men should discontinue sulfasalazine (because of reversible oliguspermia) but
not azathioprine before conception.

Question 48 A 23-year-old man presents with right-sided abdominal pain and hig- volume diarrhea
without associated passage of blood or mucous for 8 weeks. He has a history of depression and recent
traumatic fracture. His medications include sertraline and ibuprofen. A colonoscopy is arranged and the
colonic mucosa is found to be endoscopically normal. Histology shows infiltration by lymphocytes and
eosinophils in the lamina propria of the colonic epithelium and thickening of the subepithelial collagen
layer. The crypt architecture is preserved.

What is the most appropriate next step?

A. Monitor for spontaneous resolution of symptoms

B. Start oral budesonide

C. Stop ibuprofen and sertraline

D. Start sulfasalazine

E. Arrange for duodenal biopsies and exclude celiac disease

Sorry, you have selected the wrong answer.

ANSWER: C. The histology confirms the diagnosis as collagenous colitis, an inflammatory condition that
primarily affects the colon. The definite etiology is unknown, but there is evidence that certain drugs,
such as NSAIDs, SSRIs, ranitidine, aspirin, ticlopidine, simvastatin, and PPIs, are associated. Stopping the
drugs is the most appropriate initial step in management.

Around one-third of patients with celiac disease have coexistent collagenous colitis. Collagenous colitis
should be considered in patients with celiac disease who have ongoing symptoms in spite of stopping
drugs. An endoscopy and duodenal biopsy are appropriate.

Medications such as oral budesonide are indicated only if stopping drugs fails to alleviate symptoms.
Sulfasalazine is helpful in the treatment of IBD, not collagenous colitis.

Question 49 A 36-year-old man presents with bright red rectal bleeding that has occurred intermittently
over the last 2 months. His past medical history is otherwise unremarkable, and social history indicates
he was adopted at birth. He undergoes colonoscopy, and numerous (>100) small polyps are found in the
colon. Several are removed, and histological analysis confirms adenomatous histology. A clinical
diagnosis of FAP is suspected.

From an epidemiological perspective, approximately what percentage of all colorectal cancers is caused
by FAP?

A. <0.1%

B. 1%

C. 5–9%

D. 10–15%

E. >15%
Sorry, you have selected the wrong answer.

ANSWER: B. Approximately 1 in 15,000 live births will develop FAP, and currently less than 1% of all
colorectal cancers are caused by this genetic syndrome. Given the autosomal inheritance, it affects both
genders equally.

Question 50 A 42-year-old man is diagnosed with celiac disease on the basis of total villous atrophy
found on an upper endoscopy performed due to iron-deficiency anemia. In addition, his tTG antibody is
elevated. He is commenced on a gluten-free diet after meeting with a dietician.

On a follow-up visit 24 months after his initial diagnosis, his serological markers remain positive and
biopsies from the small bowel still reveal villous atrophy. He reports compliance with a gluten-free diet.

What is the most likely cause for his non-response?

A. Inadvertent gluten exposure

B. He is a genuine gluten non-responder

C. Concurrent intestinal lymphoma

D. False positivity of serum and histological testing

E. Impossible to determine – he has not been on the gluten-free diet long enough to tell

Well done, you have selected the right answer.

ANSWER: A. The most common cause of “refractory” celiac disease is dietary non-compliance or
inadvertent gluten exposure. The inadvertent gluten exposure commonly comes from processed or
imitation food products, or even make-up. Since follow-up is 24 months, healing and resolution of
serological markers would be expected. False-positive serological testing and histological analysis are
rare, as is the existence of a concurrent intestinal lymphoma.

Question 51 A 39-year-old man with known Crohn's disease presents with diarrhea. He has eight or nine
non-bloody stools daily, with a baseline of three or four. He is status post ileocecal resection and post
jejunal small-bowel resection. It is estimated that he has 105 cm of small bowel remaining. He has
recently received a course of antibiotics for a sinus infection. In addition, he reports nausea and is
currently taking ondansetron.

Which of the following factors is the least likely explanation for his diarrhea?

A. Small-intestinal bacterial overgrowth

B. Ondansetron

C. Short-gut syndrome

D. C. difficile infection due to the recent course of antibiotics

E. Bile acid malabsorption

Well done, you have selected the right answer.

ANSWER: B. The most common side effect of ondansetron is constipation. All of the other factors are
reasonable suggestions. Patients with Crohn's disease who have undergone small-bowel resection are at
risk of small-intestinal bacterial overgrowth and short-gut syndrome. Given the patient's recent
antibiotic exposure, C. difficile is also a possibility. Bile acid malabsorption is a common cause of
diarrhea, particularly in the setting of prior ileocecal resection.

Question 52 A 32-year-old woman with a 5-year history of colonic Crohn's disease on adalimumab
monotherapy presents with recurrent abdominal pain for the past 6 months. Her abdominal pain is
located in the lower abdomen and improves when she has a bowel movement. She is Carnett'
sign-negative on physical examination. She also reports fluctuating non-bloody diarrhea and
constipation. She denies any fevers, chills, or weight loss. A colonoscopy shows a normal terminal ileum
and colon. Random biopsies show inactive chronic colitis.

What is the most likely cause of her symptoms?

A. Infectious colitis

B. Crohn's disease flare

C. Irritable bowel syndrome (IBS)

D. Abdominal wall pain

E. Microscopic colitis

Sorry, you have selected the wrong answer.

ANSWER: C. Irritable bowel syndrome (IBS) is common in the background population and can occur in
upwards of 30% of inflammatory bowel disease (IBD) patients. Patients with GI symptoms should be
worked up for active inflammation, but in the absence of ulcers or other mucosal abnormalities the
diagnosis of a functional syndrome is likely, and it should be treated as such. A negative Carnett's sign
makes abdominal wall pain less likely. Biopsies have excluded microscopic colitis in this case.

Question 53 A 21-year-old man presents with a 3-month history of right lower quadrant abdominal pain
and non-bloody diarrhea. He denies fevers, chills, weight loss, nausea, or vomiting. He has had no
foreign travel, sick contact, NSAID use, or recent antibiotic use. His physical exam is notable for mild
tenderness in the right lower quadrant, without any rebound or guarding. Infectious stool studies are
negative. Colonoscopy shows mild terminal ileitis and normal colon mucosa. Biopsies show mild chronic
ileitis with crypt distortion. An MRE shows mucosal enhancement of the terminal ileum, with no other
small-bowel involvement.
What is an appropriate induction therapy?

A. Budesonide 9 mg daily

B. Azathioprine 2 mg/kg daily

C. Mesalamine 2.4 g daily

D. IV methylprednisolone 20 mg daily

E. Infliximab 10 mg/kg

Sorry, you have selected the wrong answer.

ANSWER: A. This patient has mild Crohn's disease of the terminal ileum. He has no symptoms such as
weight loss or malnutrition. Budesonide is FDA-approved to treat mild to moderate Crohn's disease of
the terminal ileum and/or right colon for 8 weeks, with possible extension to 3 months. This patient
should be started on budesonide and followed for signs or symptoms of progression of disease. Based
on the severity of illness, the patient does not require IV steroid therapy or the addition of a biologic
agent at this time. There is little evidence to support the use of mesalamine for Crohn's disease.
Azathioprine is a reasonable choice, particularly if budesonide therapy is not available, but it takes
months to begin to provide benefits.

Question 54 A 26-year-old woman in her 2nd trimester of pregnancy presents with constipation. She has
been constipated intermittently for the past 5 years and has used bisacodyl or senna as needed. She is
very concerned about taking a “safe” laxative, as her constipation has somewhat worsened as her
pregnancy has progressed.

On the basis of current recommendations, which laxative should she take?


A. Senna

B. Polyethylene glycol

C. Lubiprostone

D. Bisacodyl

E. Linaclotide

Sorry, you have selected the wrong answer.

ANSWER: B. In a comprehensive review of the safety of GI drugs in pregnancy, polyethylene glycol was
judged to be the ideal laxative, though it is classified as category C (no safety data available in animals or
humans) due to its inert properties and its minimal absorption from the GI tract. Senna is considered
safe and effective for short-term use and may be given if polyethylene glycol is ineffective. Use of
bisacodyl and lubiprostone is probably safe, though the occurrence of nausea in a substantial
percentage of lubiprostone users makes this drug less attractive, and it is far more expensive than the
other laxatives available. Linaclotide is category C, with limted data in pregnancy and currently not
recommended.

Question 55 A 28-year-old woman is referred for chronic constipation, which has worsened in the past 4
months. She complains of infrequent defecation of hard stools with moderate straining. Serum calcium,
TSH, and CBC are normal. Current medications include fluoxetine (depression) and linaclotide which was
commenced after she did not respond to polyethylene glycol and bisacodyl.

What is the most appropriate initial diagnostic approach?

A. Balloon expulsion and anorectal manometry


B. Colonoscopy

C. Colon transit study

D. Defecography

E. No testing required

Sorry, you have selected the wrong answer.

ANSWER: A. This patient has chronic constipation with no alarm features, but she has responded poorly
to osmotic, stimulant, and secretory laxatives. Therefore, the next step is to assess defecatory dynamics
using anorectal manometry and balloon expulsion. Current guidelines suggest that studies of defecation
be carried out initially, followed by colon transit study if normal. If the pelvic floor evaluation is
abnormal, treatment, usually via biofeedback should be undertaken. If the defecation disorder is
addressed and the constipation persists, colon transit studies are indicated to look for colonic inertia.
There is no indication to perform colonoscopy in a young person with chronic constipation and no alarm
features.

Question 55 A 28-year-old woman is referred for chronic constipation, which has worsened in the past 4
months. She complains of infrequent defecation of hard stools with moderate straining. Serum calcium,
TSH, and CBC are normal. Current medications include fluoxetine (depression) and linaclotide which was
commenced after she did not respond to polyethylene glycol and bisacodyl.

What is the most appropriate initial diagnostic approach?

A. Balloon expulsion and anorectal manometry

B. Colonoscopy

C. Colon transit study


D. Defecography

E. No testing required

Sorry, you have selected the wrong answer.

ANSWER: A. This patient has chronic constipation with no alarm features, but she has responded poorly
to osmotic, stimulant, and secretory laxatives. Therefore, the next step is to assess defecatory dynamics
using anorectal manometry and balloon expulsion. Current guidelines suggest that studies of defecation
be carried out initially, followed by colon transit study if normal. If the pelvic floor evaluation is
abnormal, treatment, usually via biofeedback should be undertaken. If the defecation disorder is
addressed and the constipation persists, colon transit studies are indicated to look for colonic inertia.
There is no indication to perform colonoscopy in a young person with chronic constipation and no alarm
features.

Question 56 A 52-year-old woman presents with incontinence to gas and stool for 5 years. Trials of stool
bulking agents and Kegel exercises have not been successful. She requests an operation to “take care of
my problem.” She is counseled about surgical outcomes prior to referral for further therapy.

With regard to surgery for fecal incontinence, which of the following statements is correct?

A. Fecal continence improves in approximately 70% of patients in the short term (up to ~2 years) after
surgical repair of anal sphincter defects, and this improvement is sustained over time

B. Fecal continence improves in approximately 70% of patients in the short term (up to ~2 years) after
surgical repair of anal sphincter defects, but improvement is not sustained over time

C. An artificial anal sphincter is associated with a low risk (<10%) of significant complications

D. Dynamic graciloplasty is widely used to treat fecal incontinence in the United States
E. Patients with fecal incontinence should not be offered a colostomy

Well done, you have selected the right answer.

ANSWER: B. Continence improves in approximately 70% of patients with sphincter defects following an
overlapping anterior sphincteroplasty. However, failure rates exceed 50% at 5 years after the operation.
Dynamic graciloplasty and artificial anal sphincter devices are available in a handful of centers
worldwide and are effective in selected patients. However, they are associated with considerable
morbidity (e.g., device problems, wound infection) in one-third of all patients, often requiring
reoperation. The hardware for dynamic graciloplasty is not approved for use in the United States. A
colostomy is useful for patients with severe fecal incontinence unresponsive to other measures.

Question 57 A 27-year-old woman with ileocolonic Crohn's disease on combination therapy with
azathioprine and infliximab presents to the clinic with acute-onset diarrhea. She has had between five
and eight non-bloody bowel movements a day for the past week. Associated symptoms include left
lower quadrant abdominal pain, fever, and nausea. She denies any sick contacts, recent travel, or NSAID
use.

What is the most appropriate next step?

A. A course of prednisone

B. CT of the abdomen and pelvis

C. A stool test for Clostridium difficile

D. Capsule endoscopy

E. Loperamide therapy
Sorry, you have selected the wrong answer.

ANSWER: C. A common exacerbating factor for GI symptoms in an IBD patient is concurrent C. difficile
infection. Infection can lead to increased morbidity in IBD patients and should be excluded as a cause of
acute GI symptoms before more aggressive immunosuppression is started. An empiric course of
prednisone is not recommended until an infectious exacerbation has been excluded. Imaging studies
such as CT scanning of the abdomen would be appropriate if signs of peritonism were present or if initial
investigation and management failed to reveal a cause. A capsule endoscopy is not the first-choice test
for diarrhea, and additionally Crohn's disease is a relative contraindication due to the possibility of
capsule retention at the site of a stricture. Antidiarrheal therapy is contraindicated until an infectious
cause has been excluded.

Question 58 A 48-year-old man is brought to the emergency department with syncope after 3 days of
large-volume maroon stools. He has no significant past medical history. He takes rare ibuprofen for
headache, but none in the last few weeks. He has had no alteration in his bowel habits and denies
abdominal pain. He has never had a colonoscopy. On exam, his heart rate is 115 and his blood pressure
is 95/60. He is pale and appears tired. Physical exam is unremarkable. There are no stigmata of chronic
liver disease. His admission hematocrit is 17%. He has a normal WBC, platelet count, and coagulation
studies. His BUN is 15 mg/dL and creatinine 0.7 mg/dL. He stabilizes with 2 units of packed RBCs and IV
fluids. An emergent EGD is performed and is normal. Shortly after starting the polyethylene glycol
solution in preparation for urgent colonoscopy, he experiences massive hematochezia, with hypotension
and tachycardia.

His HCT is 16%. In addition to resuscitation measures, what is the most appropriate next step?

A. Continue the colonoscopy preparation and perform colonoscopy when complete

B. Angiography

C. Capsule endoscopy

D. Emergent colectomy
E. RBC scan

Sorry, you have selected the wrong answer.

ANSWER: B. Colonoscopy is the preferred test for most patients with acute hematochezia. However, a
small proportion of patients will experience massive bleeding and cannot be stabilized for colon
preparation. Hematochezia with marked hypotension further suggests a massive upper GI bleed. In this
scenario, radiographic interventions, which can localize any bleeding source from esophagus to rectum
and do not require colon preparation, are a valuable option. There is debate regarding the need for
tagged RBC scan or CT angiography to guide subsequent angiography. Overall, the literature suggests
that these screening modalities are not useful and may delay therapeutic interventions. Colonoscopy is
generally required after radiographic diagnosis to confirm the bleeding source. Capsule endoscopy is
less helpful as an initial intervention for massive acute bleeding as it takes time to perform and is not
therapeutic. Surgery for bleeding should be avoided unless the source of bleed has been localized. Blind
emergent colectomy is associated with high morbidity and mortality and may not treat the bleeding
source (e.g., small-bowel bleeding).

Question 59 An urgent colonoscopy is being performed for acute hematochezia in a 65-year-old woman
who presented earlier the same day with hematochezia. She has no past history of GI bleed. She was
stabilized with 2 units of packed RBCs and tolerated the colon preparation. Her coagulation parameters
are all normal and she does not take any antiplatelet medications. A non-bleeding visible vessel is
identified within a broad-based diverticulum in the left colon.

What is the most appropriate next step?

A. Apply one or more endoscopic clips to the vessel and place an endoscopic tattoo near the lesion

B. Complete the colonoscopy without treatment of the lesion

C. Place an endoscopic tattoo to mark the lesion and call surgery

D. Apply thermal cautery using medium-high wattage with firm pressure until the vessel is obliterated
E. No intervention is indicated

Sorry, you have selected the wrong answer.

ANSWER: A. A number of endoscopic hemostasis techniques have been used successfully for diverticular
hemorrhage. The greatest experience is with endoscopic clips. In over 100 reported cases, hemostasis
has been achieved in more than 85%, without complications. Though successful hemostasis with
thermal cautery has also been successful in diverticular hemorrhage, there is concern regarding the
potential for thermal injury or perforation, particularly within the base of a thin-walled diverticulum. If
used in this situation, cautery should be applied with judicious power settings and pressure. Natural
history studies of diverticular bleeding stigmata suggest high rates of rebleeding and the need for
surgery without treatment. Therefore, when such lesions are identified, endoscopic treatment should be
applied. Endoscopic therapy should be attempted before recommending surgery, especially for incident
bleeding in a stable patient. Endoscopic tattooing will help localize the lesion should surgery be
necessary.

Question 60 A 48-year-old woman with Crohn's disease is being considered for vedolizumab therapy.
She has previously developed abnormal LFTs with azathioprine therapy and moderate injection
reactions to adalimumab.

Which of the following should be tested for before initiating vedolizumab therapy?

A. EBV

B. Hepatitis C

C. JCV

D. Tuberculosis
E. HPV

Well done, you have selected the right answer.

ANSWER: C. Reactivation of JCV has been observed with other integrin receptor antagonists, leading to
PML and death. While no such reports have occurred with vedolizumab, screening and counseling are
recommended if positive.

Question 61 A GI department is interested in changing its practice in order to optimize the detection of
adenomatous polyps during colonoscopy. Currently, the department guidelines recommend the use of a
split-dose 4 L polyethylene glycol bowel preparation, with a mandatory cecal withdrawal time of 8
minutes. Air insufflation is used on insertion. The current adenoma detection rate is 24%.

Which of the following measures is most likely to yield an increase in polyp detection rate at this center?

A. Using a phosphate containing bowel preparation

B. Using water instead of air during insertion of the scope

C. Increasing the cecal withdrawal time to 10 minutes

D. Having the assistant also watch the monitor during scope withdrawal

E. None of these measures is likely to increase the polyp detection rate

Sorry, you have selected the wrong answer.

ANSWER: D. The number of people observing the monitor during withdrawal has been shown to
increase polyp detection rate. There is no evidence that increasing withdrawal time beyond 8 minutes
increases yield, nor does that using water versus air during scope insertion have any effect. Phosphate
bowel preparations, which are frequently given as single doses, are inferior to split-dose polyethylene
glycol preparations.

Question 62 A 42-year-old woman presents with a 6-month history of diarrhea, weight loss of 15 lb, and
pedal edema. Evaluation for infectious causes of her diarrhea, colonoscopy with ileal evaluation and
random colon biopsies, and endoscopy with duodenal biopsies are all unremarkable. Over the past 3
weeks, she has developed increasing fatigue, weakness, and jaundice. On physical exam, she appears
acutely and chronically ill, with lethargy, mild confusion, tachycardia, muscle wasting, hair loss,
tachypnea, and an increased jugular venous pressure. Her hemoglobin is 7.8 g/dL, WBC 26 × 109/L, INR
3.1, PTT 62 seconds, total bilirubin 26 mg/dL, alkaline phosphatase 406 IU/L, and creatinine 3.2 mg/dL.
Her AST, ALT, and platelet count are normal. An abdominal ultrasound reveals ascites, and on
paracentesis the protein is 2.8 g/dL and the albumin 1.3 g/dL (serum albumin 2.6 g/dL), with 1.8 × 109/L
nucleated cells, of which 82% are polymorphonuclear leukocytes.

What is the most likely cause of her clinical presentation?

A. Adrenal insufficiency with sepsis

B. Cirrhosis with SBP

C. Thyroid storm with sepsis

D. HIV-1 infection with sepsis

E. Cholangitis with sepsis

Well done, you have selected the right answer.

ANSWER: C. Cirrhosis with sepsis and SBP does not account for the long history of diarrhea and weight
loss or the normal platelet count. Likewise, cholangitis, adrenal insufficiency, and HIV with sepsis are less
able to explain all of the findings. Thyroid storm can cause heart failure with elevated jugular venous
pulse, high protein ascites, and a high serum-to-ascites albumin gradient, as well as dyspnea and
tachycardia. Hair loss, chronic diarrhea, and jaundice can all be found in people with hyperthyroidism
complicated by thyroid storm and sepsis.

Question 63 A 52-year-old female with chronic pancreatitis presents to the emergency department with
massive hematemesis. She is hemodynamically stable. Her labs show hemoglobin 8.2 g/dL (baseline 12.1
mg/dL), platelet 240,000, WBC 4.0 cells/mm3, BUN 28 mg/dL, creatinine 2.1 mg/dL (baseline 0.9 mg/dL),
bilirubin 1.0 mg/dL, AST 21 IU/L, ALT 32 IU/L, and INR 1.0. She is resuscitated with fluids and then
undergoes an urgent upper endoscopy. Esophagus and GEJ appear normal. Large isolated fundic varices
are noted, with signs of recent bleeding.

What is the likely etiology?

A. Cirrhosis with portal hypertension

B. Splenic vein thrombosis

C. Gastric malignancy

D. Idiopathic varices

E. Budd–Chiari syndrome

Well done, you have selected the right answer.

ANSWER: B. The patient has isolated gastric varices with no evidence of esophageal varices, and this
should raise suspicion for isolated left-sided portal hypertension. She has a history of chronic
pancreatitis, so she likely has a complication of splenic venous thrombosis leading to gastric variceal
bleed. Treatment for this condition is splenectomy. The patient has normal liver tests and normal
platelet count, making cirrhosis or Budd–Chiari syndrome less likely.

Question 64 A 74-year-old woman is hospitalized with an acute stroke. After 4 days, her mental status is
poor and she is deemed at risk for aspiration, due to her inability to perform a swallow test. She is 165
cm tall and weighed 85 kg at the time of admission to hospital (BMI 31 kg/m2). A gastrostomy tube is
placed by endoscopy for longer-term tube feeding.

How should her nutritional needs be calculated?

A. From her adjusted body weight, based on height

B. From her ideal body weight, based on height

C. From her actual weight

D. Based on Stanfold thickness and ideal body weight assessment

E. Based on a review of food diary data

Sorry, you have selected the wrong answer.

ANSWER: A. Using an adjusted body weight is preferred for anthropometric assessment of patients with
BMI >30. Adjusted body weight can be calculated by averaging actual and ideal body weights.
Liver and Biliary Tract

Question 1 A 23-year-old male was involved in a major motor vehicle accident 1 year ago and had to
undergo extensive bowel resection. He has been left with just 50 cm of small bowel. He remains on TPN
for nutritional support. Routine lab testing reveals bilirubin 3.5 mg/dL, AST 120 IU/L, ALT 14 IU/L,
alkaline phosphatase 485 IU/L, and INR 1.2. Viral hepatitis serologies are negative. Ultrasound abdomen
reveals sludge in the gallbladder, with normal common bile duct.

What is the most appropriate next step?

A. Stop TPN and attempt enteral feeding

B. Decrease the lipid content of the TPN

C. Decrease the calories from dextrose in the TPN

D. Consider choline supplementation

E. All of the above

Sorry, you have selected the wrong answer.

ANSWER: E. Patients with short-bowel syndrome with less than 100 cm of bowel usually require lifelong
TPN to maintain nutrition. However, long-term TPN is associated with many complications, including
hepatic injury. Around 5–20% of patients on TPN show abnormal liver tests, usually in a cholestatic
pattern. This is believed to be due in part to the dextrose overload in the TPN, and is also influenced by
the lipid content. Any of the listed measures can be attempted in order to try and ameliorate the liver
injury.

Question 2 A 42-year-old female presents for evaluation of a painful skin rash. She has a past medical
history of diabetes mellitus and chronic hepatitis C. Her liver disease has remained compensated and
she is asymptomatic from that standpoint. Skin exam reveals a small, raised, reddish-purple rash on the
extensor surface of the lower extremities bilaterally. Laboratory studies reveal hemoglobin 12.6 g/dL,
platelets 140,000, bilirubin 2.2 mg/dL, AST 83 IU/L, ALT 96 IU/L, alkaline phosphatase 185 IU/L, INR 1.4,
creatinine 2.2 mg/dL (baseline 1.0 mg/dL), low serum complement level, and positive rheumatoid factor.

What is the best management option for her skin rash?

A. Topical corticosteroids

B. Systemic corticosteroids

C. Hepatitis C treatment

D. No treatment

E. Cyclophophamide therapy

Sorry, you have selected the wrong answer.

ANSWER: C. The patient has a history of chronic hepatitis C. The skin rash, renal failure, low complement
levels, and positive rheumatoid factor all point to a diagnosis of HCV-associated porphyria cutanea
tarda. Porphyria cutanea tarda is the most common form of porphyria and is caused by reduced activity
of uroporphyrinogen decarboxylase. The prevalence of HCV infection has been reported to be increased
in patients with porphyria cutanea tarda, and treatment of hepatitis C has been found to be helpful in
treating it.

Question 3 A 53-year-old woman is evaluated for skin itching and isolated elevation of alkaline
phosphatase. Her AMA is positive and liver biopsy is consistent with PBC with stage 2 fibrosis. She is
initiated on treatment with UDCA at 15 mg/kg, and 3 months later has improvement in alkaline
phosphatase. At 6-month outpatient follow-up, her labs show bilirubin 1.9 mg/dL, alkaline phosphatase
215 IU/L, AST 42 IU/L, ALT 35 IU/L, albumin 3.8 g/dL, INR 1.0, and platelet count 125 00. There is no
history of ascites hepatic encephalopathy or GI bleed.

What is the most appropriate next step?

A. Screen for HCC using ultrasound with AFP every 6 months

B. Perform EGD for variceal screening

C. Evaluate for transplant listing

D. Repeat liver biopsy

E. Increase the dose of UDCA to 20 mg/kg

Well done, you have selected the right answer.

ANSWER: B. Patients with PBC should undergo screening for esophageal varices even if they are not
cirrhotic as they are at risk for nodular regenerative hyperplasia (NRH), which can lead to presinusoidal
portal hypertension and cause variceal bleeding. The presence of thrombocytopenia in this patient with
non-cirrhotic liver should raise suspicion for NRH. She does not need HCC screening as she does not
have cirrhosis. Since she has compensated liver disease with a low MELD score, transplant evaluation
need not be initiated now.

Question 4 A 40-year-old woman with a past medical history of IBS and infertility is evaluated for
persistently abnormal liver tests. Routine blood testing 3 months ago revealed AST 83 IU/L and ALT 64
IU/L, with normal bilirubin, alkaline phosphatase, albumin, and INR. Hepatitis A, B, and C serologies were
negative. She was not on any medications or herbal supplements. Repeat blood testing now confirms
the persistently abnormal liver tests and reveals iron-deficiency anemia.

What is the most appropriate next step?


A. Total serum IgA and serum IgA tTG

B. Percutaneous liver biopsy

C. AMA

D. CT abdomen and pelvis

E. No further testing

Well done, you have selected the right answer.

ANSWER: A. The patient has mild persistent elevation in AST/ALT and negative viral hepatitis serologies.
Her history is significant for IBS, infertility, and iron-deficiency anemia, which are all possible
manifestations of celiac disease. Abnormal liver tests are common manifestations of celiac disease and
can be present in 30–40% of cases. The changes usually resolve on a gluten-free diet. Serologic testing
with IgA tTG has a very high sensitivity and specificity for diagnosis of celiac disease, making A the
correct choice.

Question 5 A 56-year-old Caucasian female presents for annual physical exam and is incidentally found
to have abnormal liver tests. She has a past medical history of diabetes mellitus and hypertension. She is
obese, with a BMI of 32.0. Her medications include hydrochlorothiazide 25 mg daily, metformin 500 mg
twice daily, and simvastatin 40 mg every night. She is asymptomatic and her physical exam is normal,
with no evidence for hepatosplenomegaly. Her total bilirubin is 1.5 mg/dL, AST 74 IU/L, ALT 82 IU/L, and
alkaline phosphatase 175 IU/L.

What is the most appropriate next step?

A. Decrease the dose of simvastatin to 20 mg daily


B. Stop the simvastatin

C. Continue simvastatin at current dose

D. Switch to a different statin

E. Stop the metformin

Sorry, you have selected the wrong answer.

ANSWER: C. This patient has a history of diabetes, hypertension, hyperlipidemia, and obesity, and hence
has metabolic syndrome. She is at high risk for NASH, and this is the likely explanation for her abnormal
liver tests. Statins are known to cause transient mild elevation in transaminases, but this is not an
indication to stop therapy. The incidence of true liver injury from statins is less than 1%. In this patient,
who has high cardiovascular risk, statins are more likely to be beneficial than harmful.

Question 6 A 52-year-old female with a history of hepatitis C-related cirrhosis decompensated by ascites
presents with progressive weight gain, leg swelling, and exertional dyspnea over the past 2 months. She
is compliant with salt restriction and is on furosemide 80 mg daily and spirinolactone 200 mg daily.
Physical exam reveals an elevated jugular venous pulse, holosystolic murmur along the left sternal
border (which increases with inspiration), tense ascites, and leg edema.

What might be a reason for the change in clinical symptoms?

A. Hepatopulmonary syndrome

B. Portopulmonary hypertension
C. Subacute bacterial peritonitis

D. Acute renal failure

E. Medication non-compliance

Sorry, you have selected the wrong answer.

ANSWER: B. This patient has several clinical indicators of pulmonary hypertension, including an elevated
jugular venous pulse, right-sided murmur, and weight gain. She does not have a known history of
obstructive sleep apnea or COPD, so this is likely a new onset of pulmonary hypertension.
Portopulmonary hypertension is a pulmonary arterial hypertension that is associated with portal
hypertension. Secondary causes of pulmonary hypertension need to be ruled out before this diagnosis
can be made. Patients with hepatopulmonary syndrome present with orthodeoxia, clubbing, and
hypoxia, and this clinical vignette does not have any of these features, so choice A is unlikely. Subacute
bacterial peritonitis can lead to worsening ascites but does not explain the elevated jugular venous
pulse.

Question 7 A 59-year-old man with cirrhosis secondary to chronic hepatitis C is seen in clinic for
follow-up. He reports recent worsening of ascites and decreased urine output. Physical examination is
significant for jaundice, moderate ascites, and tenderness in the right upper quadrant. Ultrasound of the
liver shows cirrhosis and a 3 cm lesion in the right hepatic lobe, raising concern for possible
hepatocellular carcinoma (HCC). Serum AFP is normal, however.

What is the most appropriate next step?

A. Repeat ultrasound in 6 months

B. Check the patient's serum CA19-9 level

C. Perform a biopsy of the liver lesion


D. Perform a contrast-enhanced CT scan

E. Perform a PET-CT scan

Well done, you have selected the right answer.

ANSWER: D. Patients with cirrhosis can develop features of hepatic decompensation when they develop
HCC. Any liver lesion noted on ultrasound in a cirrhotic patient should raise suspicion for HCC and
further cross-sectional imaging with CT or MRI should be pursued. Characteristic arterial enhancement
followed by portal venous washout on CT scan can confirm the diagnosis of HCC. Biopsies are usually not
required and should be avoided in early HCC, due to the risk of needle-track seeding. Serum CA19-9 is
used in the evaluation of cholangiocarcinoma (CCA) and not HCC.

Question 8 A 46-year-old man with alcoholic cirrhosis presents with hematemesis. Labs show
hemoglobin 8.2 g/dL, bilirubin 3.2 mg/dL, albumin 2.8 mg/dL, INR 1.2, platelets 56,000, and creatinine
1.0 mg/dL. Ultrasound shows recanalized umbilical vein with patent splenic and portal veins. Upper
endoscopy demonstrates large gastric fundal varices with active bleeding, and cyanoacrylate
(Dermabond) is injected. Bleeding initially stops, but there is recurrence of hematemesis 24 hours later.
EGD shows blood in the stomach and persistent fundal varices.

What is the most appropriate next step?

A. Splenectomy

B. Balloon tamponade

C. Repeat cyanoacrylate injection of varices

D. TIPS
E. Octreotide infusion and repeat EGD in 24 hours

Sorry, you have selected the wrong answer.

ANSWER: D. The clinical vignette describes upper GI bleed in a cirrhotic patient from gastric varices. The
varices appear to rebleed despite glue injection. Gastric varices cannot be treated with banding or
sclerotherapy, and balloon tamponade is only a temporary solution. An urgent TIPS is thus the best
means of stopping the bleeding in this patient.

Question 9 A patient with HIV and HCV co-infection is evaluated for liver transplantation. Which of the
following is an absolute contraindication to liver transplantation?

A. HIV+, CD4 count 500, negative viral load, recent bacterial peritonitis

B. HIV+, CD4 count 350, negative viral load, encephalopathy with normal LP and CT

C. HIV+, CD4 count 420, negative viral load, HCC 3.5 cm in the right lobe

D. HIV+, CD4 count 75, viral load unknown, recent pneumonia (2 weeks ago)

E. HIV+, CD4 count 420, negative viral load, HCC, two lesions of 3.0 and 1.9 cm

Sorry, you have selected the wrong answer.

ANSWER: D. Patients with HCV and HIV co-infection have a more aggressive natural history, but HIV
itself is no longer a contraindication for liver transplantation. Optimal patient selection is important to
improving outcomes in this population. Current guidelines from the American Society of Transplantation
(AST) suggest that patients with no AIDS-defining conditions should have a CD4+ T cell count >100
cells/L and those with AIDS-defining disease should have a CD4+ T cell count >200 cells/L for at least 3
months prior to transplant. The patient in option D has a CD4 count of 75 and likely has opportunistic
infection; hence he or she has an absolute contraindication for liver transplantation. All other options
include the standard indication for transplant in a well-controlled HIV patient.

Question 10 Which of the following patient or patients with malignancy should be referred for
evaluation for transplant?

A. A 54-year-old female with PBC, mild ascites, MELD 11; she was treated for axillary node-positive
breast cancer 6 months ago, has responded well to treatment, and is currently in remission

B. A 58-year-old male with alcohol-related cirrhosis, MELD 10, who has been abstinent for 5 months; a
4.2 cm HCC is diagnosed on contrast-enhanced CT

C. A 66-year-old female with NASH cirrhosis, MELD 24, and a 1.8 cm HCC diagnosed on
contrast-enhanced CT

D. A, B, and C

E. B and C

Sorry, you have selected the wrong answer.

ANSWER: E. Patient B has an HCC lesion which is within Milan criteria and hence will get MELD exception
points for liver transplant listing. He will need further evaluation and treatment for alcohol dependence,
but would benefit from treatment of his HCC lesion before it grows further. Patient C also has an HCC
that should be followed but will not get MELD exception points as the tumor is smaller than 2 cm. But
she has a high enough MELD score on the basis of her liver disease that she should be referred early.
Patient A has recent breast cancer with a high recurrence risk, but her MELD score is low, so medical
management would be appropriate.

Question 11 A 22-year-old female patient with acute liver failure (ALF) secondary to acetaminophen
overdose has rapidly progressed to hepatic encephalopahty grade IV and requires invasive intracranial
pressure (ICP) monitoring following mechanical intubation. Her platelet count is 25,000/mL, INR 2.5, and
fibrinogen level 57 mg/dL (normal >100 mg/dL).

Which of the following blood products is indicated prior to ICP monitor placement?

A. Cryoprecipitate

B. FFP

C. Platelet transfusion

D. Recombinant factor VIIa

E. All of the above

Sorry, you have selected the wrong answer.

ANSWER: E. Active bleeding and correction of coagulopathy before invasive procedures are the main
indications for blood product use in ALF. Recombinant factor VII is commonly used prior to ICP monitor
placement. However, other clotting factors need to be replaced before it can be administered (FFP to
correct INR, cryoprecipitate for severe hypofibroginemia). Most centers use a platelet count threshold
of <50,000/mL for transfusion before invasive procedures.

Question 12 The same patient as in Question 11 undergoes successful placement of a parenchymal


intracranial pressure (ICP) monitor. She has frequent episodes of ICP elevation to 25 mmHg, lasting
10–15 minutes. Her serum creatinine is 1.3 mg/dL, serum sodium 142, and urine output 50 mL/kg/hr.

What is the first-line treatment to reduce ICP?

A. Mannitol
B. Hypothermia

C. Hyperventilation

D. Hypertonic saline

E. Barbiturate-induced coma

Well done, you have selected the right answer.

ANSWER: A. Mannitol is the first-line treatment for intracranial hypertension (ICH) in acute liver failure
(ALF). Hypertonic saline, which has a similar mechanism of action, can be used if there is a
contraindication to mannitol, such as renal failure. Hypothermia and hyperventilation are second-line
treatments for ICH. Barbiturate-induced coma is usually reserved for severe ICH refractory to other
modalities.

Question 13 A 65-year old Caucasian female with alcoholic cirrhosis decompensated by moderate
ascites and hepatic encephalopathy has recently been diagnosed with HCC. She has a single large tumor
8.1 cm in maximal diameter in the right hepatic lobe, with associated right and portal vein thrombosis.
Her functional capacity is limited and she is only capable of performing activities of daily living and
self-care. Laboratory studies show sodium 129 mg/dL, potassium 3.3 mg/dL, BUN 42 mg/dL, creatinine
1.3 mg/dL, albumin 2.9 mg/dL, bilirubin 2.0 mg/dL, ALT 47 IU/L, AST 189 IU/L, alkaline phosphatase 234
IU/L, WBC 6 cells/mm3, hemoglobin 9.2 g/dL, platelet count 42,000, and AFP 4600 IU/mL.

What is the best therapeutic option?

A. Liver transplantation

B. Surgical resection
C. TACE

D. Sorafenib

E. Best supportive care

Sorry, you have selected the wrong answer.

ANSWER: D. The patient has unfortunately presented with advanced-stage HCC. Her BCLC class is Stage
C. Since she has portal hypertension and thrombocytopenia, she is not a candidate for surgical resection.
The extent of her tumor is beyond Milan criteria (one lesion between 2 to 5 cm or up to three lesions no
larger than 3 cm), so she is not eligible for liver transplantation. TACE is generally contraindicated in
patients with extensive portal venous thrombosis. The best therapeutic option is thus chemotherapy
with sorafenib, an oral multikinase kinase inhibitor that has been shown to provide a survival benefit
over best supportive care in patients with advanced HCC. Treatment with sorafenib is relatively well
tolerated; the most significant side effects are fatigue, diarrhea, hypertension, and hand–foot skin
reaction. Overall the the survival benefit is modest.

Question 14 A 49-year-old Caucasian woman with a long history of ulcerative colitis presents with
new-onset jaundice and right upper quadrant abdominal pain. The patient does not have any other
comorbidities and has an ECOG performance status of 0. An ultrasound of the liver shows a 5 cm mass in
the left lobe. CA19-9 is elevated at 4500 U/mL. Further cross-sectional imaging is pursued with
MRI/MRCP and demonstrates a heterogenous mass that is hypointense on T1 weighted images. MRCP
reveals multifocal biliary strictures with a chain-of-lakes appearance, raising the possibility of PSC. Porta
hepatic lymphadenopathy suspicious for metastatic spread is also noted. Ultrasound-guided liver biopsy
of the mass confirms the diagnosis of intrahepatic cholangiocarcinoma (CCA).

What is the most appropriate next step?

A. Surgical resection
B. Orthotopic liver transplantation

C. Transarterial chemoembolization (TACE)

D. Chemotherapy with gemcitabine and cisplatin

E. Treatment with the multikinase inhibitor sorafenib

Sorry, you have selected the wrong answer.

ANSWER: D. Patients with longstanding ulcerative colitis are at risk for PSC and hence for development
of CCA. In this patient, biopsy has confirmed the diagnosis of intrahepatic CCA, and imaging features are
consistent with this. The presence of enlarged porta hepatis lymph nodes and the high CA19-9 are likely
manifestations of substantial metastatic disease. Hence, she is not a candidate for surgical resection or
transplantation. The patient has advanced-stage intrahepatic CCA and is best treated with systemic
chemotherapy with a combination of gemcitabine and cisplatin. Sorafenib has not been FDA-approved
for the management of CCA.

Question 15 A 54-year-old male has recently been diagnosed with hemochromatosis with homozygous
mutation of the HFE gene. His lab tests at diagnosis were hemoglobin 14.5 g/dL, platelets 345,000,
creatinine 0.9 mg/dL, bilirubin 2.4 mg/dL, AST 290 IU/L, ALT 342 IU/L, INR 1.2, and ferritin 590 mg/dL. He
has been undergoing phlebotomies every week for the past 2 months. At reevaluation, his repeat labs
show hemoglobin 12.2 g/dL, bilirubin 1.5 mg/dL, AST 82 IU/L, ALT 75 IU/L, INR 1.0, and ferritin 190
mg/dL.

What is the most appropriate next step?

A. Stop phlebotomies

B. Decrease phlebotomy interval to 2–4 weeks


C. Start vitamin C supplementation and continue phlebotomies

D. Continue weekly phlebotomies

E. Increase frequency of phlebotomy

Sorry, you have selected the wrong answer.

ANSWER: B. The patient has a diagnosis of hereditary hemochromatosis with evidence of iron overload.
Patients who have evidence of iron overload should receive phlebotomy regularly until iron stores are
depleted. In the induction phase, weekly phlebotomy is performed, and the hemoglobin level is checked
before each procedure. Once the hemoglobin level is between 12 and 13 g/dL, phlebotomies can be
performed bimonthly until the ferritin level is reduced to <50 ng/mL. In the maintenance phase,
phlebotomy should be performed every 2–4 months to keep the ferritin below 50 ng/mL.

Question 16 A 35-year-old female presents with right upper quadrant pain and abdominal fullness of
acute onset. She also reports nausea, but denies vomiting, fever, or chills. She was reently diagnosed
with polycythemia vera. On physical exam, she is afebrile, tachycardic, and normotensive. She has
tender hepatomegaly with 2+ bipedal edema. Labs show hemoglobin 17.5 g/dL, creatinine 1.0 mg/dL,
total bilirubin 3.2 mg/dL, AST 350 IU/L, ALT 480 IU/L, and alkaline phosphatase 320 IU/L.

What is the most appropriate next step?

A. Ultrasound of the gallbladder

B. Ultrasound of the liver with Doppler of hepatic veins

C. Viral hepatitis serologies

D. Liver biopsy
E. Repeat lab tests in 4 weeks

Sorry, you have selected the wrong answer.

ANSWER: B. The patient is presenting with acute onset of right upper quadrant pain, new onset of liver
test abnormalities, and leg edema. This raises suspicion for a venous outflow obstruction syndrome. The
patient has known polycythemia vera, so she is at high risk for Budd–Chiari syndrome. An ultrasound
with Doppler will help evaluate for this condition. She has no prodromal features of viral hepatitis, so
that is unlikely to be the etiology, but this will have to checked after her ultrasound. A liver biopsy is
indicated only if all other tests are unhelpful and if liver test abnormalities persist.

Question 17 A 40 year old man presents for evaluation of a raised serum ferritin and an enlarged liver
uncovered by his primary care doctor on testing 6 months ago. His ALT and AST are both twice normal
with otherwise normal liver function tests. The ferritin is 2000 mg/mL. The CBC shows a leukocytosis
with a mild normochromic normocytic anemia. The CRP is mildly increased. On reviewing the history, he
is a non-drinker with no relevant family history. He complains of joint pains for several months in his
knees and wrists plus fevers. On examination, the liver is palpable with no other signs of chronic liver
disease. His BMI is 23. There is a rash over the trunk that is maculopapular and salmon colored. Testing
for the haemochromatosis gene (C282Y) is negative.

What is the most likely diagnosis?

A. Adult Still's disease

B. Haemochromatosis from another mutation

C. Chronic hepatitis C

D. Porphyria cutanea tarda

E. Fatty liver
Sorry, you have selected the wrong answer.

ANSWER: A. This patient presents with classical features of Adult Still's disease which causes a very
elevated ferritin in the absence of iron overload. A salmon coloured rash (“buzz word”), arthritis, fever
and leukocytosis make up the four major Yamaguchi criteria for Adult Stills. Haemachromatosis is
unlikely with the gene testing results. The other diseases listed do not fit the classical clinical picture
here.

Question 18 A 78-year-old man presents with new jaundice. His LFTs were normal 2 months ago, except
for an alkaline phosphatase level of 192 IU/L. He has no history of liver disease or alcohol use, and now
presents with an alkaline phosphatase of 206 IU/L, total bilirubin 3.6 mg/dL, direct bilirubin 1.6 mg/dL,
ALT 1100 IU/L, and AST 1058 IU/L. He had a syncopal episode at home 2 days ago, attributed to
paroxysmal atrial fibrillation. At present, he is comfortable and in sinus rhythm. Amiodarone was begun
yesterday. Physical examination is only remarkable for a slightly enlarged and tender liver. An abdominal
ultrasound reveals sludge in the gallbladder, but no ductal dilatation.

What is the most appropriate next step?

A. MRCP

B. Continue amiodarone

C. Doppler ultrasound of the portal vein, hepatic veins, and splenic vein

D. ERCP

E. Brain imaging
Sorry, you have selected the wrong answer.

ANSWER: B. This patient's LFTs following a syncopal episode are most consistent with acute liver injury
due to hepatic ischemia. The baseline elevated alkaline phosphatase, if hepatic, may be a result of mild
chronic passive congestive heart failure, or it could be bone in origin. This is unlikely to be biliary
obstruction or cholangitis without pain, fever, or abnormal ducts on ultrasound, so MRCP and ERCP are
not indicated. Without pain, ascites, or a history of liver disease, Doppler ultrasound is not needed.
There is nothing to suggest a cerebral event, so brain imaging is not indicated.

Question 19 A 54-year-old male presents with decompensated alcoholic cirrhosis complicated by


hepatorenal syndrome leading to acute renal failure requiring dialysis. He has developed esophageal
variceal bleeding precipitating hepatic encephalopathy. He receives cadaver donor liver transplantation
with a MELD score of 28. He is discharged without complication on day 5 after surgery. At routine
postoperative day 14 follow-up, he reports mild fever and right upper quadrant abdominal pain of 1
day's duration. Cholestatic liver profile and a fluid collection near the porta hepatis are seen on CT scan.
CT-guided aspiration confirms an aseptic biloma. Doppler ultrasound of the hepatic artery is normal.

What is the most appropriate next step?

A. ERC to assess for biliary anastomotic leak and subsequent stent placement

B. Relisting for transplantation

C. Percutaneous liver biopsy

D. Urgent surgical revision

E. Percutaneous drain placement in the fluid collection

Sorry, you have selected the wrong answer.


ANSWER: A. The patient's clinical presentation is highly suggestive of a bile leak, which is usually
common at the site of biliary anastomosis. An ERC is helpful in diagnosing the site of leak and allows
therapeutic placement of a biliary stent. The patient's ultrasound Doppler does not show hepatic artery
thrombosis and there is no clinical evidence for graft failure, so there is no need for relisting or to pursue
surgical revision.

Question 20 The same patient in Question 19 develops nausea, vomiting, and diarrhea 4 months after
transplant. Stool studies are negative. Transaminases are elevated and CMV viremia is detected.

What is the most appropriate next step?

A. Repeat stool studies

B. Increase immunosuppression levels

C. Perform EGD and colonoscopy with mucosal biopsies

D. Begin empirical antifungal treatment

E. Refer for surgical evaluation.

Sorry, you have selected the wrong answer.

ANSWER: C. The patient is at high risk for CMV gastroenteritis since he is within 6 months of transplant
and has developed CMV viremia. To confirm a histological diagnosis of this he will need an endoscopy
and colonoscopy with multiple biopsies.

Question 21 The same patient in Question 19 calls 1 year after treatment to inquire about
recommendations for annual cancer screening.
Which of the following statements is true?

A. Patients with cholestatic disease are at a greater risk of developing skin cancers post transplant

B. All liver transplant patients require annual colonoscopy

C. Patients with alcoholic cirrhosis are at an increased risk of certain cancers post transplant

D. Thymoglobulin induction does not increase the lymphoma risk

E. Screen for prostate cancer in those men over age 20 years.

Sorry, you have selected the wrong answer.

ANSWER: C. There is an increased risk of cancers following liver transplant. Oropharyngeal cancers are
notably increased in former alcoholic patients. Prostate cancer screening is recommended in those men
over aged 40. Post transplant lymphoproliferative disorder (associated with Ebstein Barr virus) is linked
to aggressive post transplant immunosuppression including thymoglobulin. Colonoscopy is generally
recommended every 3-5 years over aged 50 (unless there is PSC and UC when annually colonoscopy is
recommended). There is a 20 fold increased risk of skin cancer post transplant but cholestatic liver
disease is NOT a risk factor.

Question 22 A 48-year-old man is admitted to the hospital with right upper quadrant tenderness,
weakness, and altered mental status. Physical examination is significant for multiple spider angiomata of
the abdomen, palmar erythema, tense distended abdomen that is tender to palpation, and asterixis.
Labs are significant for AST 368 IU/L, ALT 146 IU/L, total bilirubin 6.2 mg/dL, prothrombin time 34
seconds (control 12 seconds), and albumin 1.3 g/dL. The patient is febrile to 38.2 °C and is hypotensive
at 92/66 mmHg. Cultures are drawn, and empiric antibiotics and lactulose are initiated. His friend who
brought him to the hospital reports that over the last 12 months the alcohol consumption has been
excessive.

What is the most appropriate next step?


A. Start methylprednisolone

B. Withhold steroids, due to possible infection, and treat with N-acetylcysteine

C. Start parenteral nutrition containing medium-chain fatty acids.

D. Order urgent endoscopy

E. Initiate assessment for liver transplantation

Sorry, you have selected the wrong answer.

ANSWER: A. The patient's clinical presentation is consistent with acute alcoholic hepatitis.
Hyperbilirubinemia, prolongation of prothrombin time (PT), and hypoalbuminemia are prognostic
indicators of severe alcoholic hepatitis. Maddrey's Discriminant Function, which is calculated as 4.6×(PT
patient − PT control)+total bilirubin (mg/dL), exceeds 32 in this patient, and mortality in this group
excess 50%. Steroids have been shown to benefit patients in this subgroup, so methylprednisolone is the
correct choice. Fever and leukocytosis can be a part of the inflammatory response to alcoholic hepatitis
and do not essentially indicate infection. If no clinical evidence of overt infection is present, steroids can
be initiated. There are no overt signs of bleeding, so endoscopy is not needed. The patient has not been
treated for his alcohol dependence so is not a candidate for liver transplantation at this point.

Question 23 A patient with PSC cirrhosis undergoes screening upper endoscopy and is noted to have
large esophageal varices with no signs of recent bleeding.

What is the most appropriate next step?

A. Repeat endoscopy in 6 months


B. Repeat endoscopy in 1 year

C. Esophageal variceal banding

D. Initiation of beta-blocker therapy for primary prophylaxis

E. TIPS

Well done, you have selected the right answer.

ANSWER: D. Primary prophylaxis with beta-blockers is indicated to decrease the risk of variceal bleeding.
In patients who are not eligible for or do not tolerate beta-blocker therapy, variceal banding can be used
for prophylaxis. Both strategies have been shown to be more effective than no treatment in preventing
variceal bleeding, but beta-blockers are considered first-line as they are non-invasive. If the patient were
found to have small varices then they could be placed on surveillance endoscopy without initiating
medical management. TIPS is only used in the management of refractory variceal bleeding that is
unresponsive to other therapies.

Question 24 A 39-year-old female is evaluated in the emergency room for right-sided abdominal pain of
24 hours duration. She reports no past medical history, and was well prior to presentation. She denies
dysuria and has no recent fevers. Her last menstrual period was 1 week ago. The patient's only
medication is an oral contraceptive, which she has been using for most of the past 21 years. CBC, serum
chemistries, and urinalysis and urine pregnancy test are negative. Examination is negative, except for
mild tenderness to palpation in the right upper quadrant. Pulse and blood pressure are normal. An
ultrasound suggests a vague liver hypodensity in the right hepatic lobe. Subsequent MRI with
gadolinium reveals an arterially enhancing 6 cm slightly heterogeneous mass in the right hepatic lobe,
part of which extends to the liver capsule, causing mild bulging.

What is the most appropriate next step?

A. The patient should be reassured that the lesion appears to be a benign FNH and no further workup is
needed
B. The patient should be referred to a surgeon for resection of the mass

C. The patient should be referred to a liver transplant center for possible transplantation

D. The patient should be instructed to discontinue her oral contraceptive

E. The patient should be followed, with repeat imaging in 6 months

Sorry, you have selected the wrong answer.

ANSWER: B. The vignette describes a solitary hepatic mass in a woman who uses contraception. MRI
features are consistent with a hepatic adenoma: this is usually a benign tumor, but lesions larger than 5
cm are at risk for malignant transformation to HCC. This adenoma is also causing significant abdominal
pain. For these reasons, she needs to be evaluated for surgical resection. The imaging features are not
consistent with FNH, which usually has a central scar. Though discontinuing oral contraceptives has been
shown to lead to tumor shrinkage, this patient has a large tumor, so this approach is not appropriate in
this case.

Question 25 A 63-year-old male with cirrhosis attributed to hepatitis C and prior alcohol abuse
undergoes triphasic abdominal CT imaging for liver cancer surveillance. His calculated MELD score is 9.
He has mild ascites and encephalopathy, for which he is on diuretics and lactulose, respectively. He has
not been evaluated for liver transplantation. AFP is 11. His last liver imaging by ultrasound 6 months ago
showed a cirrhotic liver but no solid masses. CT abdomen now reveals a new 1.2 cm arterially enhancing
mass in segment 3 of the left hepatic lobe. The lesion does not demonstrate “washout” on delayed
phases, nor is a pseudocapsule seen.

What is the most appropriate next step?

A. Refer the patient for transhepatic chemoembolization or RFA of the mass


B. Perform a biopsy of the mass

C. Perform MRI of the abdomen

D. Refer the patient to a surgeon for resection of the mass

E. Repeat imaging in 3–6 months

Sorry, you have selected the wrong answer.

ANSWER: C. Liver masses in a cirrhotic liver are highly likely to represent HCC. This lesion shows arterial
enhancement, which is characteristic of HCC, but delayed washout is also required to make a radiologic
diagnosis. Indeterminate lesions need to be evaluated by another cross-sectional contrast imaging
modality to help clarify the radiological features. So MRI is the most appropriate next step. If the lesion
remains indeterminate on both modalities then a biopsy can be considered. The patient has
decompensated cirrhosis and hence is not a good candidate for resection. TACE or RFA can be pursued
once the diagnosis is established.

Question 26 A 55-year-old man presents to his primary care physician for his annual checkup. He has not
seen his primary care provider since receiving his referral for his colonoscopy examination 5 years ago,
which showed no polyps. His past medical history is insignificant, except for a tonsillectomy at the age of
4. He is currently on no medications and has no known allergies. The patient admits to a 20-year
pack-per-day smoking history, though he quit 15 years ago. He drinks one glass of wine for dinner and
has never done any illicit drugs. The patient is currently married to his partner of 30 years and had not
had other sexual contacts. His family history is significant for his father dying of colon cancer at 78 and
his mother still living with type 2 diabetes, which she developed at age 70. Review of systems is
insignificant. The chart states that the patient received his last tetanus shot at 49.

Assuming his birth date is January 1, 1959, which of the following screening exams is appropriate for this
patient?

A. Abdominal ultrasound to evaluate for an aortic aneurism


B. Colonoscopy

C. Hepatitis B screening

D. Hepatitis C screening

E. EGD

Sorry, you have selected the wrong answer.

ANSWER: D. The US Preventive Services Task Force (USPSTF) strongly recommends that individuals born
between the years 1945 and 1965 be offered a one-time screening examination for hepatitis C. Though
the patient is male and a previous smoker, he is still under the age of 65, making choice A incorrect.
Choice B is incorrect because current guidelines suggest a patient should undergo a colonoscopy every
10 years if in the average-risk category. Finally, hepatitis B screening is not recommended routinely
unless a patient has high-risk behavior, such as IV drug use or multiple sexual partners. EGD is not
indicated for routine screening, and the patient has no longstanding history of reflux symptoms.

Question 27 A 73-year-old man presents to a travel clinic for evaluation prior to a planned trip to India in
2 weeks' time. He is travelling to a rural part of southern India and will live in a small village, eating
native food.

How should this patient be advised regarding vaccination for hepatitis A?

A. Vaccination is not necessary

B. The patient should receive two doses of inactivated hepatitis A vaccine prior to travel

C. The patient can travel after one dose of hepatitis A vaccine


D. The patient should receive the first dose of hepatitis A vaccine and a dose of immunoglobulin

E. The patient should only receive immunoglobulin

Well done, you have selected the right answer.

ANSWER: D. All eligible persons travelling to endemic regions need to be vaccinated for hepatitis A. If
travel is more than 4 weeks away then active immunization with hepatitis A vaccination alone will
suffice. In older people who are travelling within 2 weeks of evaluation, a combination of active and
passive immunization is recommended to prevent hepatitis A infection. Given this patient's age, D is the
best option.

Question 28 Which of the following patients are at increased risk of developing fulminant hepatic failure
following acute hepatitis A infection?

A. Women during pregnancy

B. Patients with chronic HCV infection

C. Patients exposed to hepatitis A during infancy

D. Patients with chronic renal failure

E. Patients with previous exposure to HBV who developed anti-HBSAg

Sorry, you have selected the wrong answer.

ANSWER: B. Hepatitis A is one of the most frequently encountered infectious liver diseases in children
worldwide. The disease is usually mild and self-limited, and complications are not common. But rarely,
hepatitis A can cause acute fulminant liver failure, which is a severe and life-threatening condition.
Patients with underlying chronic liver disease are at higher risk for this complication, which is why they
need to be vaccinated for hepatitis A. Hepatitis A is not transmitted transplacentally, and hence
pregnant women and infants are not at risk for this complication.

Question 29 A 50-year-old man was diagnosed 1 year ago with chronic hepatitis C genotype 1a. A liver
biopsy revealed stage 4 fibrosis, but he remained compensated, with no evidence of ascites or hepatic
encephalopathy. He was treated with combination sofosbuvir and ledipasvir for 12 weeks and has
attained SVR.

What test is needed for long-term follow up?

A. EGD every year for variceal screening

B. Ultrasound every 6 months for HCC screening

C. HCV RNA every year to screen for recurrence

D. Liver tests every year

E. None of the above

Well done, you have selected the right answer.

ANSWER: B. The patient's pretreatment liver biopsy demonstrates cirrhotic-stage disease. Once SVR is
achieved, patients are not at risk for HCV recurrence, and hence HCV RNA need not be followed. But
cirrhotic patients remain at risk for liver cancer and so should continue surveillance with ultrasound.

Question 30 An 82-year-old woman is seen for 8 months of pedal edema. Her examination is remarkable
for anasarca and pedal edema, with a normal cardiovascular exam. She has no history of cardiac,
hepatic, or renal disease. Her hemoglobin is 8.7 g/dL, WBC normal, platelet count 152 × 109/L, alkaline
phosphatase 725 IU/L, total bilirubin 1.2 mg/dL, ALT 59 U/L, albumin 1.9 g/dL, INR 1.1, calcium 11.2
mg/dL, and creatinine 2.2 mg/dL. Abdominal ultrasound shows mild hepatomegaly, normal spleen,
normal bile ducts, and trace ascites.

What is the most appropriate next step?

A. ACE level

B. Liver biopsy

C. Serum protein electrophoresis

D. MRCP

E. Antinuclear and smooth-muscle antibodies

Sorry, you have selected the wrong answer.

ANSWER: C. This patient most likely has an infiltrative disorder of the liver, with hyperlipidemia,
anasarca, anemia, and renal dysfunction, and amylodosis is much more likely than sarcoidosis (option A)
or autoimmune liver disease (option E). Ductal disease is also less likely (option D). A liver biopsy is
invasive and likely unnecessary to diagnose this patient's cholestatic liver disorder.

Question 31 A 65-year-old male with a past medical history of alcoholic cirrhosis decompensated by
variceal bleeding presents with worsening dyspnea over the past several weeks. He denies a history of
smoking or previous lung disease. He reports that the dyspnea gets better if he lies down to rest. On
exam, he has mild conjunctival icterus, multiple spider angiomas on the chest and abdomen, pan-digital
clubbing, splenomegaly, and 2+ bipedal pitting edema. Chest X-ray appears normal. Arterial blood gas
shows pH 7.42, pO2 59, and pCO2 38.
What is the most appropriate next step?

A. Dobutamine stress test

B. CT chest with PE protocol

C. Pulmonary function tests with 6-minute walk

D. Bronchoscopy with bronchial lavage

E. Transthoracic echocardiogram with shunt study

Sorry, you have selected the wrong answer.

ANSWER: E. This cirrhotic patient has clinical features of platypnea, clubbing, and hypoxia, raising
suspicion for hepatopulmonary syndrome. Hepatopulmonary syndrome is more common in patients
who have a history of esophageal varices, and these patients also tend to have spider angiomas. A PaO2
of <80 mmHg indicates impaired oxygenation. A contrast echo shows evidence of a right-to-left shunt.
With an intrapulmonary shunt, contrast appears in the left heart three to six beats after its appearance
in the right heart, unlike with an intracardiac shunt, where the contrast appears in the left side within
three beats.

Question 32 A 25-year-old female comes to the emergency department with a 10-day history of
progressive malaise and fatigue, anorexia, and abdominal discomfort. The patient uses IV heroin on a
daily basis and drinks approximately two to three cans of beer daily. Physical examination demonstrates
jaundice and a tender, enlarged liver. There are no other stigmata of chronic liver disease. Laboratory
studies show CBC normal, INR 1.1, serum alkaline phosphatase 120 IU/L, serum AST 1250 IU/L, serum
ALT 2120 IU/L, serum total bilirubin 3.5 mg/dL, serum albumin 3.6 g/dL, HBSAg negative, IgM anti-HAV
negative, anti-HCV negative, and IgM anti-HBc negative.

Which test is most likely to establish the diagnosis?


A. IgG anti-Hepatitis A virus

B. IgG anti-Hepatitis B core antigen

C. HCV RNA

D. Anti-Hepatitis B surface antibody

E. Anti mitochondrial antibody (AMA) titer

Sorry, you have selected the wrong answer.

ANSWER: C. The patient presents with clinical features consistent with acute viral hepatitis. Since IgM
anti-HAV and IgM anti-HBc are negative, it is unlikely to be acute hepatitis A or B. The patient's anti-HCV
antibody is negative, but this can take several weeks to appear following exposure. HCV RNA appears
early, so checking this will help evaluate for acute hepatitis C. AMA is a marker of PBC, which does not
present with acute hepatitis.

Question 33 A 36-year-old primigravida at 34 weeks' gestation presents with anorexia, nausea, and
epigastric pain for 2 days. Physical examination shows jaundice, tachycardia, BP 140/95, and a distended
pregnant abdomen, but no hepatosplenomegaly or ascites. She is drowsy, with slow mentation. Labs
show total bilirubin 13.6 mg/dL, direct bilirubin 8.7 mg/dL, AST 580 IU/mL, ALT 753 IU/mL, hemoglobin
10.5 g/dL, WBC 27,000, platelets 105,000, INR 3.2, glucose 90 mg/dL, and creatinine 1.6 mg/dL.

What is the most likely clinical diagnosis?

A. HELLP syndrome

B. Fulminant viral hepatitis


C. Acute fatty liver of pregnancy (AFLP)

D. Budd–Chiari syndrome

E. Intrahepatic cholestasis of pregnancy

Sorry, you have selected the wrong answer.

ANSWER: C. AFLP is a rare disease that exclusively occurs in the third trimester of pregnancy. If not
recognized and managed early, it can be associated with high maternal mortality. Laboratory features
include modest transaminitis, elevated bilirubin, and low platelets. Those with more severe disease also
develop coagulopathy and hypoglycemia. Patients with HELLP syndrome have hemolysis and anemia,
and the patient in this vignette has normal hemoglobin. Patients with fulminant viral hepatitis usually
have AST/ALT in the 1000s, unlike this patient. Patients with intrahepatic cholestasis of pregnancy
usually do not develop features of significant transaminitis or liver failure.

Question 34 A 72-year-old female is referred for evaluation of abnormal liver tests. She has a history of
stage III RCC post resection, with neoadjuvant chemoradiotherapy 6 months ago. Her past medical
history includes hypertension treated with metoprolol, hyperlipidemia treated with simvastatin, and
diabetes mellitus treated with metformin. She has been on stable doses of these three medications for
4–5 years. She was recently diagnosed with maxillary sinusitis and was treated with a 7-day course of
amoxicillin/clavulanic acid. On routine testing, she was found to have bilirubin 1.5 mg/dL, AST 82 IU/L,
ALT 74 IU/L, alkaline phosphatase 300 IU/L, and INR 1.0. Liver tests were completely normal 2 months
ago. She is asymptomatic. Ultrasound of the abdomen is normal.

What is the likely explanation for her acute change in liver tests?

A. Liver metastases from renal cancer

B. Amoxicillin/clavulanic acid-induced liver injury


C. Statin-induced liver injury

D. Cholelithiasis

E. NASH

Sorry, you have selected the wrong answer.

ANSWER: B. The patient has recent onset of mild transaminitis and normal liver synthetic function. Liver
imaging does not show any evidence of metastatic disease. Statins can cause mild elevations in AST and
ALT, but the values usually normalize within the first several weeks. This patient has been on statins for
many years, so they are unlikely to be the culprit. No steatosis is noted on liver ultrasound, so NASH is
unlikely. The best explanation for the patient's abnormal liver enzymes is the recent use of
amoxicillin/clavulanic acid. This drug-induced liver injury will have to be closely followed to ensure that
it resolves.

Question 35 A 33-year-old male with a history of IV drug use is hospitalized with acute hepatitis and
jaundice. Laboratory studies demonstrate AST 1500 IU/L, ALT 2000 IU/L, bilirubin 12 mg/dL, INR 1.5, and
normal ALP, albumin, and bilirubin. An acute hepatitis panel shows anti-HAV IGM negative, HBSAg
positive, anti-core IgM positive, and anti-HCV negative. Over the next 2 days, serum ALT declines to 120
IU/L and INR normalizes. The patient is discharged, but he returns 2 days later with confusion. Serum
AST and ALT are now up to 1500 and 2000 IU/mL, respectively, INR is 4.1, and bilirubin in 22 mg/dL.

Which statement is true of this patient's condition?

A. He likely has a relapsing form of acute HAV

B. He likely has acute liver failure (ALF)secondary to acute HBV

C. He likely has ALF secondary to acute HCV superimposed upon chronic HBV infection
D. He likely has ALF secondary to superinfection with HDV on top of chronic HBV

E. He likely has ALF secondary to co-infection with HBV and HDV

Well done, you have selected the right answer.

ANSWER: D. The patient has clinical features suggestive of acute hepatitis B, since his HBSAg and
anti-core IgM are positive. His liver enzymes appear to improve and INR normalizes only to worsen after
a few days: superinfection of HBV infection with hepatitis D infection can lead to such worsening in
clinical condition, making D the right choice. The patient's anti-HAV IGM is negative, so this is not acute
hepatitis A. Co-infection of HBV and HDV does not explain the recurrence of symptoms after initial
improvement.

Question 36 Which of the following is NOT a risk factor for the development of severe cholangitis?

A. Obesity

B. Advanced age

C. History of smoking

D. Impacted common bile duct stone

Sorry, you have selected the wrong answer.

ANSWER: A. Acute cholangitis is a relatively common complication of gallstone disease that may present
with abdominal pain, fever, and jaundice. Broad-spectrum, parenteral antibiotics are effective and
typically allow the patient to be stabilized prior to ERCP for biliary decompression. However, a minority
of patients with acute cholangitis develop severe cholangitis, with a high risk of biliary sepsis and poor
outcomes.
Advanced age, history of smoking, and the presence of an impacted stone in the common bile duct are
all risk factors for the development of severe cholangitis. Severe cholangitis, often called “acute
suppurative cholangitis,” results from accumulation of pus in the bile duct, which increases biliary
pressures and predisposes to biliary sepsis. Patients at risk for acute suppurative cholangitis should be
considered for urgent ERCP. Obesity is a risk factor for gallstone disease but is not associated specifically
with severe cholangitis.

Question 37 A 50-year-old alcoholic with previous variceal bleed presents with tense ascites. Vitals
reveal no fever. Exam shows tense ascites. Paracentesis shows a total protein of 1.2 mg/dL (serum
albumin 2.4 mg/dL) and WBC 550 cells/mm3 (20% PMN). After 24 hours, one ascitic culture bottle grows
E. coli.

The patient remains well. What is the diagnosis?

A. SBP

B. Culture-negative neutrocytic ascites

C. Culture contamination

D. Non-neutrocytic bacteriascites

E. Secondary bacterial peritonitis

Sorry, you have selected the wrong answer.

ANSWER: D. SBP can be a serious complication of cirrhosis, with high morbidity and mortality.
Interpretation of ascitic fluid cell count and cultures in order to identify SBP and its variants is important
in determining treatment. If ascitic fluid PMN count is >250 and fluid cultures are positive then the
diagnosis is SBP. If the count is normal but cultures are positive then the diagnosis is non-neutrocytic
bacteriascites. If such cultures are positive with a significant delay following initial fluid collection and if
atypical organisms are grown then culture contamination should be considered. If ascitic fluid PMN
count is >250 but cultures are negative then the diagnosis is culture-negative neutrocytic ascites. This
patient has a normal PMN count (110) but a positive ascitic fluid culture, so the diagnosis is
non-neutrocytic bacteriascites.

Question 38 A 45-year-old man with decompensated cirrhosis presents with increasing confusion and
difficulty sleeping through the night. His physician recently prescribed zolpidem 5 mg QHS for insomnia,
but this did not help.

What is the most appropriate next step?

A. Increase zolpidem to 10 mg QHS

B. Change to diazepam 5 mg QHS

C. Start neomycin 1 g QID

D. Stop zolpidem, start lactulose

E. Don't change medications, just monitor

Well done, you have selected the right answer.

ANSWER: D. Reversal of the sleep cycle is one of the early manifestations of hepatic encephalopathy.
Hepatic encephalopathy patients are very sensitive to sedatives, and these medications can worsen the
condition. The recent addition of zolpidem is the likely culprit here, making the patient more confused.
It needs to be stopped, and the patient treated with lactulose, which is the first-line therapy for hepatic
encephalopathy.
Question 39 What is the most common complication of symptomatic gallstone disease?

A. Cholecystocholedochal fistula

B. Gallstone ileus

C. Acute cholecystitis

D. Mirizzi's syndrome

Sorry, you have selected the wrong answer.

ANSWER: C. Acute cholecystitis is the most common complication of gallstones, occurring in 20% of
patients with symptomatic cholelithiasis. Acute cholecystitis occurs when a gallstone obstructs the cystic
duct and results in acute gallbladder inflammation. Cholecystocholedochal fistula and Mirizzi's
syndrome are less frequent complications, occurring in approximately 1% of patients undergoing
surgery for gallbladder disease. Mirizzi's syndrome occurs when an impacted gallstone in the cystic duct
or the neck of the gallbladder compresses the adjacent bile duct, resulting in complete or partial
obstruction of the common hepatic duct. A cholecystocholedochal fistula can occur when a gallstone
impacted in the neck of the gallbladder erodes into the bile duct. Gallstone ileus occurs when a large
migrated gallstone becomes impacted in the small bowel, most often at the terminal ileum and ileocecal
valve. While generally uncommon, gallstone ileus is a significant cause of small-bowel obstruction in the
elderly.

Question 40 A 25-year-old female who is 32 weeks pregnant complains of worsening fatigue and reflux
symptoms. She also reports recent onset of mild leg edema, which improves with rest. On physical
exam, she is mildly tachypneic, afebrile, and normotensive. She is noted to have palmar erythema,
spider angiomas, and bipedal 1+ pitting edema. Her primary care physician orders lab tests as part of the
evaluation for fatigue, which show hemoglobin 10.5 g/dL, MCV 90, platelet count 155,000, total bilirubin
1.2 mg/dL, AST 23 IU/L, ALT 21 IU/L, alkaline phosphatase 240 IU/L, albumin 3.2 gm/dL, and INR 1.0.
Hepatitis B and C serologies are negative.
What is the most appropriate next step?

A. No further evaluation is needed

B. Perform upper endoscopy to evaluate GERD

C. Perform ultrasound of the liver with Doppler

D. Check AMA

E. Perform CT venogram of the abdomen

Well done, you have selected the right answer.

ANSWER: A. All these findings are physiological changes of pregnancy, so no further evaluation is
required. Leg edema in pregnancy can result from compression of IVC by the gravid uterus. Palmar
erythema and spider angiomas reflect the hyperestrogenic state. Alkaline phosphatase is elevated in
pregnancy, as it is released from the placenta. The anemia, thrombocytopenia, and hypoalbuminemia
can all be related to dilutional changes.

Question 41 A healthy 21-year-old female college student presents to the clinic with 5 days of fatigue,
malaise, sore throat, and intermittent fever. On physical exam, there is tender lymphadenopathy of the
cervical, axillary, and inguinal lymph nodes, as well as mildly tender hepatomegaly and splenomegaly;
otherwise, her exam is unremarkable. Laboratory tests reveal elevated ALT and AST, with preserved liver
synthetic function and moderate lymphocytosis.

Which is the most likely cause of the clinical presentation?

A. HSV infection
B. HAV infection

C. Chronic HCV infection

D. Acute HBV infection

E. EBV infection

Sorry, you have selected the wrong answer.

ANSWER: E. There are several clinical features in the vignette which point to a likely diagnosis of
infectious mononucleosis, including the presence of sore throat, hepatosplenomegaly,
lymphadenopathy, and lymphocytosis. The patient will likely need to also be evaluated for acute
hepatitis A, B, and C, but IgM EBV antibody or a monotest will help make the diagnosis of EBV infection.

Question 42 A healthy 47-year-old woman recently immigrated to the United States from central
Mexico. She is diagnosed with acute HEV, with positive HEV IgM and detectable HEV RNA in serum.

What is the most likely form of transmission in this case?

A. Blood transfusion

B. Consumption of raw pork

C. Exposure to a sick contact

D. Consumption of contaminated drinking water


E. Vertical transmission

Sorry, you have selected the wrong answer.

ANSWER: D. The patient has been diagnosed with acute hepatitis E and likely acquired it in Mexico.
Hepatitis E, like hepatitis A, is transmitted feco-orally. Hence, the likely source of HEV infection is
drinking fecally contaminated water. In developed countries, pigs can be infected with HEV and serve as
a reservoir. Eating undercooked or raw pork has been documented as leading to transmission of HEV,
but this is rare. Choice D is the more likely explanation here.

Question 43 A 48-year-old Caucasian female presents with severe abdominal pain, fevers, and gross
hematuria. Laboratory studies demonstrate serum ALT 223 IU/L, serum albumin 3.5 mg/dL, and
creatinine 3.0 mg/dL. Serum ALP and bilirubin are normal. A CT scan of the abdomen demonstrates
numerous aneurysms of the renal and mesenteric arteries.

Which statement is most likely to be correct?

A. Hepatitis C RNA will be positive

B. Anti-Hepatitis A will be positive

C. The patient can be treated with peginterferon

D. Anti-Hepatitis B e antigen will be positive

E. Anti-Hepatitis B surface Ab will be positive


Sorry, you have selected the wrong answer.

ANSWER: D. The patient has acute renal failure with hematuria, and imaging features showing
aneurysms in medium-sized vessels are consistent with polyarteritis nodosa (PAN). PAN is known to be
associated with hepatitis B, and hence choice D is correct. This is an indication for treatment of hepatitis
B, but peginterferon is not the drug of choice: direct antivirals like entecavir and tenofovir are preferred.

Question 44 A 25-year-old Hispanic female is diagnosed with hepatitis B during her antenatal visit. She is
referred to discuss treatment options. She has been asymptomatic from a hepatitis B standpoint. Her
lab studies show hemoglobin 10.5 g/dL, platelets 185,000, bilirubin 1.2 mg/dL, AST 63 IU/L, ALT 46 IU/L,
alkaline phosphatase 180 IU/L, INR 1.0, HBsAg positive, HBsAb negative, HBeAg positive, HBeAb
negative, HBV DNA 3.5 million IU/mL.

What is the best treatment strategy to prevent vertical transmission to her unborn child?

A. Give HBV vaccine to mother and child

B. Treat mother with oral antiviral therapy and give HBV vaccine to child

C. Treat mother with oral antiviral therapy and give HBIG and HBV vaccine to child

D. Give HBV vaccine to mother and HBIG to child

E. Treat mother with oral antiviral therapy and give HBV vaccine to child

Sorry, you have selected the wrong answer.

ANSWER: C. This patient has chronic active hepatitis, given that she has abnormal ALT, positive eAg, and
high viral load. The risk of vertical transmission to the fetus in her scenario is as high as 90%. Provision of
active and passive immunization to the infant after delivery has been shown to have >95% efficacy in
decreasing the risk of vertical transmission. Additionally, antiviral therapy should be offered to mothers
with high HBV DNA levels, since it can further reduce the risk of perinatal transmission. Tenofovir and
lamivudine are available for therapy; the former is preferred as it is associated with a lower rate of
resistance.

Question 45 What is the best test for staging of gallbladder cancers?

A. CT scan

B. EUS

C. Transabdominal ultrasound

D. MRCP

Sorry, you have selected the wrong answer.

ANSWER: B. EUS can be used to assess the depth of tumor invasion into the wall of the gallbladder and
to define lymph node involvement. EUS can also be used to guide FNA in order to obtain bile for
cytologic analysis or direct sampling of a mass.

Question 46 A 45-year-old male with history of alcoholic cirrhosis decompensated by ascites and hepatic
encephalopathy attends for a follow-up visit. He complains of refractory ascites despite medical
management. He is on a low-sodium diet (90 meq/day), along with furosemide 40 mg daily and
spirinolactone 100 mg oral daily. The patient confirms that he has been compliant with sodium
restriction and medication intake. He has required a therapeutic paracentesis every 2–3 weeks, with 5–6
L of fluid removed each time. He denies fever, abdominal pain, or GI bleed. His exam reveals a thin,
poorly nourished adult male with a large, protuberant abdomen. He has multiple spider nevi and palmar
erythema. No asterixis is noted. Abdomen exam reveals the presence of free fluid with positive fluid
thrill. Labs show sodium 132 mg/dL, potassium 4.0 mg/dL, creatinine 1.3 mg/dL, and 24-hour urine
sodium 250 meq.

What is the most appropriate next step?


A. Increase the furosemide to 80 mg once daily

B. Increase the spirinolactone to 100 mg po daily

C. Increase both furosemide and spirinolactone

D. Administer IV albumin

E. Refer to a dietician

Sorry, you have selected the wrong answer.

ANSWER: E. The vignette describes a cirrhotic with refractory ascites that requires repeated
paracentesis despite diuretic therapy. Patients with ascites are counseled to adopt a low-sodium diet
(<2 g/day), and most find this difficult to follow. A 24-hour urine sodium is useful in determining
whether patients are compliant with salt restriction. This patient has a 24-hour urine sodium of 250
mmol, which is high (usual <90 mmol/day). Hence, he should be referred to a dietician for education
about dietary sodium restriction: a crucial part of the management of ascites.

Question 47 A 45-year-old male with a history of alcoholic cirrhosis is admitted to the hospital for
hematemesis. His hemoglobin on arrival is 6.0 g/dL and his platelet count in 75,000. He is transfused 1
unit of PRBC and started on IV octreotide and IV PPI, along with prophylactic IV antibiotics. He
undergoes an emergent upper endoscopy once he is clinically stable, and large gastric varices are found
without any evidence for esophageal varices or gastroesophageal varices.

What is the most appropriate next step?

A. Perform interventional radiology to perform emergent TIPS


B. Undertake general surgery consultation

C. Perform cyanoacrylate glue injection

D. Monitor clinically and transfuse as appropriate

E. Consider colonoscopy for evaluation of lower GI bleed

Well done, you have selected the right answer.

ANSWER: C. Gastric varices are the second most common cause of upper GI bleed in patients with portal
hypertension, after esophageal varices. Bleeding from gastric varices is generally more severe, is
associated with higher transfusion requirements, and has higher morbidity and mortality than bleeding
from esophageal varices. Traditional endoscopic therapies like sclerosing therapy and variceal banding
are not indicated for management of gastric varices as they are associated with a high rebleeding rate.
In centers with experienced endoscopists, cyanacrylate glue injection has been proposed to be the first
step in the management of these varices. A significant portosystemic shunt will have to be ruled out
before glue injection. If glue injection fails, then TIPS is the next best option.

Question 48 A 42-year-old female undergoes an abdominal and pelvic ultrasound, ordered by her
primary care provider because of recurrent UTIs. She is well between episodes, and denies abdominal
pain. The patient used oral contraceptives intermittently in the past for a total duration of less than 5
years. She had a hysterectomy 5 years ago for menorrhagia refractory to other therapies. CBC and liver
blood tests are normal. Ultrasound shows no renal or pelvic pathology, but a 4 cm hypodense lesion is
found incidentally in the left hepatic lobe. Subsequent MRI shows the lesion to be a homogeneous 4.2
mass, mildly hyperintense on T2-weighted images, with a central pale region of 9 mm suggestive of a
central scar.

What is the most appropriate next step?

A. The lesion is a hemangioma; no further imaging or follow-up is necessary


B. The lesion is an FNH; 1- and 2-year follow-up MRI is reasonable to ensure stability

C. The lesion is a hepatic adenoma; refer to a surgeon for resection

D. Biopsy the lesion now to confirm benign diagnosis

E. The lesion is HCC; list the patient for liver transplantation

Sorry, you have selected the wrong answer.

ANSWER: B. FNH is a benign liver lesion with no malignant potential. Radiologically, FNH appears to be
hyperintense on T2-weighted images, with a central scar. The lesion described in the vignette has
characteristic radiologic features of FNH and hence a biopsy is not required.

Question 49 A 45-year-old female complains of right upper quadrant abdominal pain for the past 6
months. The pain appears to worsen after eating and lasts for 2–3 hours. Her primary care physician
orders an ultrasound of her abdomen, which reveals normal gallbladder wall thickness, no
pericholecystic fluid, and no stones in the gallbladder. A 1.2 cm gallbladder polyp is noted in the fundus.

What is the most appropriate next step for the gallbladder polyp?

A. Laparoscopic cholecystectomy

B. Repeat ultrasound in 6 months

C. No surgery is needed: reassure the patient

D. CT scan abdomen
E. ERCP with sphincterotomy

Well done, you have selected the right answer.

ANSWER: A. Gallbladder polyps are benign ingrowths in the wall of the gallbladder that are usually
incidentally found on imaging studies. Patients who have gallbladder polyps and stones should undergo
cholecystectomy as they are at high risk for malignancy. In patients with PSC and a gallbladder polyp, a
cutoff size of 6–8 mm is used to recommend cholecystectomy. Patients with a gallbladder polyp >2 cm
should generally undergo preoperative staging with a CT scan and EUS prior to cholecystectomy.
Gallbladder polyps between 1 and 2 cm in size also have high malignant potential but can be removed
with laparoscopic cholecystectomy; no preoperative staging is required.

Question 50 A 50-year-old Caucasian female presents with fever, fatigue, right upper quadrant pain, and
jaundice. Physical exam reveals conjunctival icterus, jaundice, and mild tender hepatomegaly. She is not
taking any medications, denies alcohol use, and has recently traveled. Laboratory data show normal
CBC, normal electrolyte panel, and normal renal function. Liver tests are abnormal: total bilirubin 7.8
mg/dL, AST 2300 IU/L, ALT 4500 IU/L, alkaline phosphatase 345 IU/L, INR 1.5, and albumin 3.9 gm/dL.
HAV IgM is positive, and hepatitis B and C antibodies are negative.

The patient is diagnosed with acute hepatitis A and advised conservative management. She shows initial
clinical improvement over a period of 4 weeks, but begins to notice worsening jaundice and recurrence
of right upper quadrant pain. Repeat lab studies at 4 weeks reveal total bilirubin 13.5 mg/dL, AST 1825
IU/L, ALT 950 IU/L, alkaline phosphatase 250 IU/L, INR 1.3, and albumin 3.3 gm/dL. Repeat hepatitis
serologies again demonstrate a positive HAV IgM and with negative HBV and HCV antibodies. Serum
protein electrophoresis reveals an increased gamma-globulin ratio, but ANA is negative. Abdominal
ultrasound reveals a normal gallbladder with no stones and a common bile duct diameter of 3 mm.

Over the next 4 weeks, the patient remains quite fatigued and develops pruritus. At around 3 months
after initial presentation, her jaundice begins to subside, and repeat labs show total bilirubin 1.1 mg/dL,
AST 84 IU/L, and ALT 60 IU/L.

What is a possible explanation for her clinical presentation?


A. Prolonged cholestasis with hepatitis A

B. Overlap with Autoimmune hepatitis (AIH)

C. Recurrence of hepatitis A

D. Chronic liver disease from hepatitis A

E. Overlap with Primary biliary cirrhosis (PBC)

Sorry, you have selected the wrong answer.

ANSWER: A. Hepatitis A usually runs a benign course and completely resolves in 4–6 weeks, but this
patient has persistent cholestasis for 4 months. Cholestatic hepatitis A is a rare presentation of hepatitis
A in which the liver enzymes remain persistently elevated and the IgM anti-HAV remains positive for
more than 3 months. No treatment is necessary as it resolves spontaneously.

Question 51 Which of the following diagnostic tests does not need to be obtained in a 22-year-old
female with abdominal pain, ALF, ALT of 1000 IU/ml, and a urine drug screen positive for opiates?

A. Acetaminophen level

B. Ceruloplasmin level

C. Liver biopsy

D. HBsAg, anti-HBc, anti-HAV


E. Liver ultrasound with Doppler

Sorry, you have selected the wrong answer.

ANSWER: C. Liver biopsy in acute liver failure (ALF) is usually reserved to confirm autoimmune hepatitis
(AIH) or exclude liver transplant candidacy (malignancy) in select cases. A serum acetaminophen level is
usually detectable in subjects presenting within 24 hours of a single-time-point acetaminophen
overdose but may be low or undetectable in patients with non-intentional overdose. A serum
ceruloplasmin, 24-hour urine copper, and slit-lamp exam are used to evaluate for Wilson's disease. A
liver ultrasound is used to exclude Budd–Chiari and liver masses.

Question 52 A 52-year-old female who underwent living donor liver transplantation for PBC 1 year ago
presents for routine follow-up. She reports mild itching but otherwise feels well. She is on tacrolimus
immunosuppression at 1 mg p.o. b.i.d. Physical exam is unremarkable. Lab tests show hemoglobin 13.5
g/dL, platelets 250,000, bilirubin 5.1 mg/dL, ALT 122 IU/L, AST 110 IU/L, alkaline phosphatase 425 IU/L,
INR 1.0, and tacrolimus 8.7 mg/dL. Ultrasound of the liver demonstrates normal liver parenchyma, with
no intrahepatic biliary dilatation. Doppler shows normal flow through the hepatic artery, with normal
resistive indices. The common bile duct appears normal.

What is the most appropriate next step?

A. ERCP

B. Liver biopsy

C. Increase of the dose of tacrolimus

D. MRCP

E. UDCA therapy
Sorry, you have selected the wrong answer.

ANSWER: A. Patients with living donor transplantation are at high risk for biliary complication, such as
biliary strictures. This patient's clinical picture is suggestive of a cholestatic process, but ultrasound does
not show intra- or extrahepatic dilation. In liver transplant patients, biliary obstruction does not lead to
significant biliary dilation, and hence ERCP is the optimal test, as it can serve as both a diagnostic and a
therapeutic modality. The clinical picture is not consistent with recurrent PBC, so UDCA is unlikely to be
helpful. The tacrolimus level appears adequate, so the patient should continue the current dose.

They end

You might also like